Bio 011 Quiz, Exam and Textbook questions

Réussis tes devoirs et examens dès maintenant avec Quizwiz!

What ratio of offspring would result from a cross between a white-eyed male and a female that is heterozygous for red eye color?

Half of the female offspring would be heterozygous (XWXw) with red eyes, and half would be homozygous recessive (XwXw) with white eyes. Half of the male offspring would be hemizygous dominant (XWY) withe red yes, and half would be hemizygous recessive (XwY) with white eyes.

Both of the regular intravenous solutions administered in medicine, normal saline and lactated Ringer's solution, are isotonic. Why is this important?

Injection of isotonic solutions ensures that there will be no perturbation of the osmotic balance, and no water taken from tissues or added to them from the blood.

If the rate of replication in a particular prokaryote is 900 nucleotides per second, how long would it take 1.2 million base pair genomes to make two copies?

1333 seconds or 22.2 minutes.

If the nucleolus were not able to carry out its function, what other cellular organelles would be affected? A. Both cytosolic ribosomes and rough endoplasmic reticulum B. Smooth endoplasmic reticulum C. Rough endoplasmic reticulum D. Nucleus E. Cytosolic Ribosomes

A

In a cell that makes alot of steroid hormones, which of the following structures would take up much of the cell? A. Smooth endoplasmic reticulum B. Rough endoplasmic reticulum C. peroxisomes D. cell wall E. lysosomes

A

Which of the following mutations is least likely to be a frame-shift mutation? A. An insertion of six nucleotides B. A deletion of four nucleotides C. An insertion of two nucleotides D. A deletion of eight nucleotides E. An insertion of ten nucleotides

A

Explain the difference between a proto-oncogene and a tumor-suppressor gene.

A proto-oncogene is a segment of DNA that codes for one of the positive cell cycle regulators. If that gene becomes mutated so that it produces a hyperactivated protein product, it is considered an oncogene. A tumor suppressor gene is a segment of DNA that codes for one of the negative cell cycle regulators. If that gene becomes mutated so that the protein product becomes less active, the cell cycle will run unchecked. A single oncogene can initiate abnormal cell divisions; however, tumor suppressors lose their effectiveness only when both copies of the gene are damaged.

In what situation(s) would the use of a scanning electron microscope be ideal, and why?

A scanning electron microscope would be ideal when you want to view the minute details of a cell's surface, because its beam of electrons moves back and forth over the surface to convey the image.

Explain in your own words the difference between a spontaneous reaction and one that occurs instantaneously, and what causes this difference.

A spontaneous reaction is one that has a negative ∆G and thus releases energy. However, a spontaneous reaction need not occur quickly or suddenly like an instantaneous reaction. It may occur over long periods due to a large energy of activation, which prevents the reaction from occurring quickly.

Calculate the probability of selecting a heart or a face card from a standard deck of cards. Is this outcome more or less likely than selecting a heart suit face card?

A standard deck of cards contains 52 cards, 13 of which are hearts and 12 of which are face cards. Heart suit OR face card: This calculation requires the sum rule since there are multiple pathways to successfully pulling a desired card. 13/52 + 12/52 = 25/52 = 0.48 The probability of selecting a heart suit or a face card is significantly more likely than the probability of selecting a heart suit face card (3/52 = 5.8).

Cells grown in the laboratory are mixed with a dye molecule that is unable to pass through the plasma membrane. If a ligand is added to the cells, observations show that the dye enters the cells. What type of receptor did the ligand bind to on the cell surface?

An ion channel receptor opened up a pore in the membrane, which allowed the ionic dye to move into the cell.

The diagnosis of Down syndrome is made by examining the individual's: A. gametes B. karyotype. C. chromatin. D. nucleosomes. E. kinetochores.

B

Which of the following correctly explains the process of random alignment of homologous chromosomes as well as when and why this happens during meiosis? A. In metaphase I, duplicated copies of maternal and paternal homologous chromatids assort themselves across the center of the cell according to whether they are maternally or paternally derived. As the homologous chromosomes are pulled apart in anaphase I, any combination of maternal and paternal chromosomes will move toward each pole resulting in unique gametes that lead to new combinations of traits. B. In metaphase I, duplicated copies of maternal and paternal homologous chromosomes randomly line up across the center of the cell. There is an equal chance that the maternally (or paternally) derived chromosomes will be facing either pole. This alignment occurs differently in nearly every meiotic division. As the homologous chromosomes are pulled apart in anaphase I, any combination of maternal and paternal chromosomes will move toward each pole resulting in unique gametes that lead to new combinations of traits. C. In metaphase II, duplicated copies of maternal and paternal homologous chromatids assort themselves across the center of the cell according to whether they are maternally or paternally derived. As the homologous chromosomes are pulled apart in anaphase II, specific combinations of chromosome appear according to whether they are maternal or paternal resulting in gametes that lead to new combinations of traits during fertilization. D. In prophase I, duplicated copies of maternal and paternal homologous chromatids assort themselves across the center of the cell according to whether they are maternally or paternally derived. As the homologous chromosomes are pulled apart in telophase I, any combination of maternal and paternal chromosomes will move toward each pole resulting in unique gametes that lead to new combinations of traits. E. In metaphase II, duplicated copies of maternal and paternal homologous chromosomes randomly line up across the center of the cell. There is an equal chance that the maternally (or paternally) derived chromosomes will be facing either pole. This alignment occurs differently in nearly every meiotic division. As the homologous chromosomes are pulled apart in anaphase II, any combination of maternal and paternal chromosomes will move toward each pole resulting in unique gametes that lead to new combinations of traits.

B

Quinolone antibiotics treat bacterial infections by blocking the activity of topoisomerase. Why does this treatment work? Explain what occurs at the molecular level.

Bacteria treated with quinolones will no longer be able to replicate their DNA. Topoisomerase relieves the excess DNA supercoiling that occurs ahead of the replication fork as DNA is unwound for replication. If topoisomerase is inhibited, DNA helicase will only be able to unwind the DNA for a short stretch before the supercoiling becomes too overwound for replication to continue.

What are the roles of ATP and NADPH in photosynthesis?

Both of these molecules carry energy; in the case of NADPH, it has reducing power that is used to fuel the process of making carbohydrate molecules in light-independent reactions.

Discuss how buffers help prevent drastic swings in pH.

Buffers absorb the free hydrogen ions and hydroxide ions that result from chemical reactions. Because they can bond these ions, they prevent increases or decreases in pH. An example of a buffer system is the bicarbonate system in the human body. This system is able to absorb hydrogen and hydroxide ions to prevent changes in pH and keep cells functioning properly.

A scientist records that the movement of molecules from one animal cell to another is occurring in a fragment of a millisecond. Which of the following is responsible for rapid movement of these molecules? A. Microfilaments B. Mitochondria C. Gap Junctions D. Plasmodesmata E. ATP

C

Chromosomes contain large amounts of interacting proteins known as: Answers: A. chromatins B. nucleosomes C. histones. D. protein hormones. E. cyclins.

C

How many neutrons do carbon-12 and carbon-13 have, respectively?

C-12 has 6 neutrons and C-13 has 7 neutrons

What makes ionic bonds different from covalent bonds?

Covalent bonds involve sharing electrons between atoms while ionic bonds form between ions of opposite charges. In covalent bonds, atoms share electrons, whereas in ionic bonds atoms transfer electrons. The reaction components of covalent bonds are electrically neutral, whereas for ionic bonds they are both charged.

New neural pathways are constantly being built. When this happens two neurons, that never connected before, begin to communicate with one another. This process of building the neural pathway underlies everything from learning how to walk to memory for bio11 concepts. Which of the following best describes how two neurons can meet to form a new connection that helps you remember important motor skills, events or information? A. Integral membrane proteins receive ligands that initiate signal transduction pathways. B. Membrane proteins are anchored to cytoskeleton to provide stability and adhesion. C. Proteins with the cell membrane act as enzymes that initiate intracellular responses D. Cell adhesion molecules that are proteins embedded in the cell membrane guide cell migration to the point of contact and then other membrane proteins begin forming intracellular synaptic junctions with the new neighboring cell. E. All of these allow new cells to form new connections.

D

Which of the following is NOT a function of a lipid? A. Insulation B. Building cell membranes C. Long term energy stores D. catalyze reactions E. Building hormones

D

Describe the structure and complementary base pairing of DNA.

DNA has two strands in anti-parallel orientation. The sugar-phosphate linkages form a backbone on the outside, and the bases are paired on the inside: A with T, and G with C, like rungs on a spiral ladder.

What is the initial source of electrons for the chloroplast electron transport chain? A. ATP B. NADPH C. NAD D. RuBisCo E. Water

E

Which of the following correctly describes how buffers help prevent drastic swings in pH? A. Buffers bond these H+ or OH- ions to prevent increases or decreases in pH. B. Buffer systems, such as the bicarbonate system in the human body, can absorb hydrogen and hydroxide ions to prevent changes in pH and keep cells functioning properly. C. Buffers prevent large changes in pH to avoid acidic or basic solutions from occurring. D. Buffers absorb the free hydrogen ions and hydroxide ions that result from chemical reactions. E. All of the statements here are correct statements about how buffers work

E

Which of the following correctly describes the products of dehydration synthesis? A. polymers B. amino acids C. monomers D. water E. A and D are correct

E

Explain in your own words how enzyme feedback inhibition benefits a cell.

Feedback inhibition allows cells to control the amounts of metabolic products produced. If there is too much of a particular product relative to the cell's needs, feedback inhibition effectively causes the cell to decrease production of that particular product. In general, this reduces the production of superfluous products and conserves energy, maximizing energy efficiency.

What is the primary difference between fermentation and anaerobic respiration?

Fermentation uses glycolysis only. Anaerobic respiration uses all three parts of cellular respiration, including the parts in the mitochondria like the citric acid cycle and electron transport; it also uses a different final electron acceptor instead of oxygen gas.

Normal gamete cell produced in meiosis II

G and H

An atom may give, take, or share electrons with another atom to achieve a full valence shell, the most stable electron configuration. Looking at this figure, how many electrons do elements in group 1 need to lose in order to achieve a stable electron configuration? How many electrons do elements in groups 14 and 17 need to gain to achieve a stable configuration?

Group 1 elements would need to lose their lone valence electron in their outermost shell to achieve a stable electron configuration. Group 14 elements would need to gain 4 electrons to achieve a stable configuration. Group 17 elements would need to gain 1 electron to achieve a stable configuration.

A frameshift mutation that results in the insertion of three nucleotides is often less deleterious than a mutation that results in the insertion of one nucleotide. Why?

If three nucleotides are added, one additional amino acid will be incorporated into the protein chain, but the reading frame won't shift.

What is the function of the fused kinetochore found on sister chromatids in prometaphase I?

In metaphase I, the homologous chromosomes line up at the metaphase plate. In anaphase I, the homologous chromosomes are pulled apart and move to opposite poles. Sister chromatids are not separated until meiosis II. The fused kinetochore formed during meiosis I ensures that each spindle microtubule that binds to the tetrad will attach to both sister chromatids.

If a peripheral membrane protein were synthesized in the lumen (inside) of the ER, would it end up on the inside or outside of the plasma membrane?

It would end up on the outside; after the vesicle passes through the Golgi apparatus and fuses with the plasma membrane, it turns inside out.

Why might negative feedback mechanisms be more common than positive feedback mechanisms in living cells?

Negative feedback mechanisms actually control a process; it can turn it off, whereas positive feedback accelerates the process, allowing the cell no control over it. Negative feedback naturally maintains homeostasis, whereas positive feedback drives the system away from equilibrium.

Explain how single nucleotide changes can have vastly different effects on protein function.

Nucleotide changes in the third position of codons may not change the amino acid and would have no effect on the protein. Other nucleotide changes that change important amino acids or create or delete start or stop codons would have severe effects on the amino acid sequence of the protein.

What is the role of a primer in DNA replication? What would happen if you forgot to add a primer in a tube containing the reaction mix for a DNA sequencing reaction?

Primer provides a 3′-OH group for DNA pol to start adding nucleotides. There would be no reaction in the tube without a primer, and no bands would be visible on the electrophoresis.

Why were radioactive sulfur and phosphorous used to label bacteriophage in Hershey and Chase's experiments?

Sulfur is an element found in proteins and phosphorus is a component of nucleic acids.

Eukaryotic chromosomes are thousands of times longer than a typical cell. Explain how chromosomes can fit inside a eukaryotic nucleus.

The DNA double helix is wrapped around histone proteins to form structures called nucleosomes. Nucleosomes and the linker DNA in between them are coiled into a 30-nm fiber. During cell division, chromatin is further condensed by packing proteins.

Where does the cell get energy for active transport processes?

The cell harvests energy from ATP produced by its own metabolism to power active transport processes, such as the activity of pumps.

Why are energy carriers thought of as either "full" or "empty"?

The energy carriers that move from the light-dependent reaction to the light-independent one are "full" because they bring energy. After the energy is released, the "empty" energy carriers return to the light-dependent reaction to obtain more energy. There is not much actual movement involved. Both ATP and NADPH are produced in the stroma where they are also used and reconverted into ADP, Pi, and NADP+.

Would you describe metabolic pathways as inherently wasteful or inherently economical? Why?

They are very economical. The substrates, intermediates, and products move between pathways and do so in response to finely tuned feedback inhibition loops that keep metabolism balanced overall. Intermediates in one pathway may occur in another, and they can move from one pathway to another fluidly in response to the needs of the cell.

How does the structure of a plasmodesma differ from that of a gap junction?

They differ because plant cell walls are rigid. Plasmodesmata, which a plant cell needs for transportation and communication, are able to allow movement of really large molecules. Gap junctions are necessary in animal cells for transportation and communication.

A scientist observes that a cell has an RNA polymerase deficiency that prevents it from making proteins. Describe three additional observations that would together support the conclusion that a defect in RNA polymerase I activity, and not problems with the other polymerases, causes the defect.

To determine that a RNA polymerase I mutation or deficiency is causing the defect in protein production, the scientist would need to make observations that provide evidence that RNA polymerases II and III are working in the cell. The observations eliminating RNA polymerase II as the defect could include: · Transcription of mRNAs in the nucleus; · Presence of processed mRNAs in the cytoplasm The observations eliminating RNA polymerase III could include: · Isolation of small nuclear RNAs from the cell; · Isolation of microRNAs from the cell; · Transcription of 5S rRNA in the nucleus; · Presence of tRNAs in the cytoplasm The observations implicating RNA polymerase I could include: · A lack of functional ribosomes in the cytoplasm (RNA polymerase I or III); · A lack of RNA polymerase I protein; RNA polymerase I protein is non-functional

Metabotropic receptors use second messenger systems that amplify the initial signal input

True

Plant cells are eukaryotes

True

Secondary Active Transport uses energy derived from an electrochemical gradient

True

The following reaction is an endergonic reaction: ADP + Pi → ATP

True

How would you perform a reciprocal cross for the characteristic of stem height in the garden pea?

Two sets of P0 parents would be used. In the first cross, pollen would be transferred from a true-breeding tall plant to the stigma of a true-breeding dwarf plant. In the second cross, pollen would be transferred from a true-breeding dwarf plant to the stigma of a true-breeding tall plant. For each cross, F1 and F2 offspring would be analyzed to determine if offspring traits were affected according to which parent donated each trait.

If a mutation occurs so that a fungus is no longer able to produce a minus mating type, will it still be able to reproduce?

Yes, it will be able to reproduce asexually.

What compound receives electrons from NADH? a. FMN b. ubiquinone c. cytochrome c1 d. oxygen

a

List and briefly describe the three processes that lead to variation in offspring with the same parents.

a. Crossover occurs in prophase I between nonsister homologous chromosomes. Segments of DNA are exchanged between maternally derived and paternally derived chromosomes, and new gene combinations are formed. b. Random alignment during metaphase I leads to gametes that have a mixture of maternal and paternal chromosomes. c. Fertilization is random, in that any two gametes can fuse.

1) A scientist compares the plasma membrane composition of an animal from the Mediterranean coast with one from the Mojave Desert. Which hypothesis is most likely to be correct? a) The cells from the Mediterranean coast animal will have more fluid plasma membranes b) The cells from the Mojave Desert animal will have a higher cholesterol concentration in the plasma membranes. c) The cells' plasma membranes will be indistinguishable. d) The cells from the Mediterranean coast animal will have a higher glycoprotein content, while the cells from the Mojave Desert animal will have a higher lipoprotein content.

b

1. A scientist identifies a pre-mRNA with the following structure. What is the predicted size of the corresponding mature mRNA in base pairs (bp), excluding the 5' cap and 3' poly-A tail? a. 220bp b. 295bp c. 140bp d. 435bp

b

In which compartment of the plant cell do the light-independent reactions of photosynthesis take place? a. thylakoid b. stroma c. outer membrane d. mesophyll

b

Which of the following have the ability to disassemble and reform quickly? a. microfilaments and intermediate filaments b. microfilaments and microtubules c. intermediate filaments and microtubules d. only intermediate filaments

b

Which of the following is the best way to judge the relative activation energies between two given chemical reactions? a. Compare the ∆G values between the two reactions b. Compare their reaction rates c. Compare their ideal environmental conditions d. Compare the spontaneity between the two reactions

b

9. Which order of molecular conversions is correct for the Calvin cycle? a. RuBP + G3P → 3-PGA → sugar b. RuBisCO → CO2 → RuBP → G3P c. RuBP + CO2 → [RuBisCO] 3-PGA → G3P d. CO2 → 3-PGA → RuBP → G3P

c

A diploid cell has_______ the number of chromosomes as a haploid cell. a. one-fourth b. half c. twice d. four times

c

Which of the following statements is false? a. Molecules with the formulas CH3CH2COOH and C3H6O2 could be structural isomers b. Molecules must have a double bond to be cis-trans isomers c. To be enantiomers, a molecule must have at least three different atoms or groups connected to a central carbon d. To be enantiomers, a molecule must have at least four different atoms or groups connected to a central carbon

c

Which type of molecule acts as a signaling molecule in yeasts? a. steroid b. autoinducer c. mating factor d. second messenger

c

A cleavage furrow separates the daughter cells

cytokenisis

A doctor is researching new ways to treat biofilms on artificial joints. Which approach would best help prevent bacterial colonization of the medical implants? a. Increase antibiotic dosing b. Create implants with rougher surfaces c. Vaccinate patients against all pathogenic bacteria d. Inhibit quorum sensing

d

A mutated gene that codes for an altered version of Cdk that is active in the absence of cyclin is a(n) _____. a. kinase inhibitor b. tumor suppressor gene c. proto-oncogene d. oncogene

d

Where do DAG and IP3 originate? a. They are formed by phosphorylation of cAMP b. They are ligands expressed by signaling cells c. They are hormones that diffuse through the plasma membrane to stimulate protein production d. They are the cleavage products of the inositol phospholipid, PIP2

d

Which characteristic of a phospholipid contributes to the fluidity of the membrane? a) its head b) cholesterol c) a saturated fatty acid tail d) double bonds in the fatty acid tail

d

Receptors in the human body respond to five types of stimuli in the external and internal environment. Receptors in major vessels leaving the heart constantly monitor blood pressure in order to instigate a correctional response if something should go terribly wrong with circulation. What type of receptor is used to monitor blood pressure?

mechanoreceptors

Sister chromatids are attached to a spindle fiber originating from opposite poles of the cell.

metaphase

A mistake in mitosis that results in somatic cells with abnormal addition of a chromosome.

none of these are correct

In the reaction below, the reaction between A and C is an example of a ______________ reaction whereas the reaction between B and D is an example of a ______________ reaction. 2 NaCl (A) + F2 (B) → Cl2 (C) + 2 NaF (D)

oxidative; reduction

The inhibition of growth is a cardinal symptom of zinc deficiency. In animals fed a zinc-inadequate diet, growth is reduced within 4-5 d. Results show that reduced energy intake is not the limiting factor in growth because force-feeding a zinc-inadequate diet to animals fails to maintain growth. Zinc was found to participate in the regulation of cell proliferation needed for growth via actions on DNA. This is an example of Zinc acting as a __________________________ during translation.

promoter

What peptide chain would be produced from this sequence? 5' AGATGCCGCACTGAC 3' 3' TCTACGGCGTGACTG 5'

serine, threonine, alanine

Given the mRNA as in (a), what changes to the sequence of the resulting polypeptide would occur if the underlined G/C base pair at position 52 was changed to a C/G sequence? (this is a fill in the blank, the answer is just a few words, not whole sentences of paragraphs)

silent mutation

Chromosomes arrive at opposite poles and begin to decondense

telophase

What is the difference between intracellular signaling and intercellular signaling?

Intracellular signaling occurs within a cell, and intercellular signaling occurs between cells.

Which of the following describes the tonicity of the cell in A? A: 0.15 M NaCl (inside the cell), 0.15 M NaCl (outside the cell) B: 0.15 M NaCl (inside the cell), Pure H2O (outside the cell) C: 0.15 M NaCl (inside the cell), 1.5 M NaCl (outside the cell)

Isotonic

On a hot, dry day, plants close their stomata to conserve water. What impact will this have on photosynthesis?

Levels of carbon dioxide (a necessary photosynthetic substrate) will immediately fall. As a result, the rate of photosynthesis will be inhibited.

Receptors in the human body respond to five types of stimuli in the external and internal environment. When a mountain climber summits Mount Everest at 30,000 feet, receptors in the brain are constantly monitoring the drop in oxygen levels that is occurring. What type of receptor is used to monitor hypoxic (i.e. low oxygen) conditions?

Ligand Chemical

Which of the following statements about Okazaki fragments is true? A. They occur because DNA polymerase operates in only one direction along a strand of DNA. B. They act as a primer that initiates DNA replication. C. If they did not exist, the ends of chromosomes would get shorter with every replication. D. It they did not exist, bases would pair with their complementary bases. E. They reduce the mutation rate during DNA replication.

A

Describe the pathway of electron transfer from photosystem II to photosystem I in light-dependent reactions.

A photon of light hits an antenna molecule in photosystem II, and the energy released by it travels through other antenna molecules to the reaction center. The energy causes an electron to leave a molecule of chlorophyll a to a primary electron acceptor protein. The electron travels through the electron transport chain and is accepted by a pigment molecule in photosystem I.

In what situation(s) would a transmission electron microscope be ideal, and why?

A transmission electron microscope would be ideal for viewing the cell's internal structures, because many of the internal structures have membranes that are not visible by the light microscope.

The next nucleotide to be added to a growing strand is dCTP. Which of the letters below describes where it will bind to the growing chain of DNA? A. A B. B C. C D. D E. E F. F G. G H. H I. I J. J

A. -OH (hydroxyl) group

How did the scientific community learn that DNA replication takes place in a semi-conservative fashion?

Meselson's experiments with E. coli grown in 15N deduced this finding.

Describe how the duplicated bacterial chromosomes are distributed into new daughter cells without the direction of the mitotic spindle.

As the chromosome is being duplicated, each origin moves away from the starting point of replication. The chromosomes are attached to the cell membrane via proteins; the growth of the membrane as the cell elongates aids in their movement.

Curly wing is recessive to straight wing in Drosophila. In a cross between true-breeding curly- and straight-winged parental flies, there are 480 flies in the F2 generation. How many F2 progeny should have curly wings? A. 25 B. 75 C. 120 D. 240 E. 360

C

During asexual reproduction, the genetic material of the parent is passed on to the offspring by: A. homologous pairing. B. meiosis and fertilization. C. mitosis and cytokinesis. D. karyotyping. E. chiasmata.

C

Discuss how degeneracy of the genetic code makes cells more robust to mutations.

Codons that specify the same amino acid typically only differ by one nucleotide. In addition, amino acids with chemically similar side chains are encoded by similar codons. This nuance of the genetic code ensures that a single-nucleotide substitution mutation might either specify the same amino acid and have no effect, or may specify a similar amino acid, preventing the protein from being rendered completely nonfunctional.

In mitotic metaphase, a cell with eight sister chromatids has _______ centromeres. A. zero B. one C. two D. four E. eight

D

You isolate a cell strain in which the joining of Okazaki fragments is impaired and suspect that a mutation has occurred in an enzyme found at the replication fork. Which enzyme is most likely to be mutated?

DNA ligase, as this enzyme joins together Okazaki fragments.

Explain at least three functions that lipids serve in plants and/or animals.

Fat serves as a valuable way for animals to store energy. It can also provide insulation. Waxes can protect plant leaves and mammalian fur from getting wet. Phospholipids and steroids are important components of animal cell membranes, as well as plant, fungal, and bacterial membranes.

Why are fatty acids better than glycogen for storing large amounts of chemical energy?

Fats have a higher energy density than carbohydrates (averaging 9 kcal/gram versus 4.3 kcal/gram, respectively). Thus, on a per gram basis, more energy can be stored in fats than can be stored in carbohydrates. Additionally, fats are packaged into spherical globules to minimize interactions with the water-based plasma membrane, while glycogen is a large branched carbohydrate that cannot be compacted for storage.

Imagine an elaborate ant farm with tunnels and passageways through the sand where ants live in a large community. Now imagine that an earthquake shook the ground and demolished the ant farm. In which of these two scenarios, before or after the earthquake, was the ant farm system in a state of higher or lower entropy?

The ant farm had lower entropy before the earthquake because it was a highly ordered system. After the earthquake, the system became much more disordered and had higher entropy.

Why would an individual with a mutation that prevented the formation of recombination nodules be considered less fit than other members of its species?

The chromosomes of the individual cannot cross over during meiosis if the individual cannot make recombination nodules. This limits the genetic diversity of the individual's gametes to what occurs during independent assortment, with all daughter cells receiving complete maternal or paternal chromatids. An individual who cannot produce diverse offspring is considered less fit than individuals who do produce diverse offspring.

Name the common components of eukaryotic cell division and binary fission.

The common components of eukaryotic cell division and binary fission are DNA duplication, segregation of duplicated chromosomes, and division of the cytoplasmic contents.

Why is it advantageous for the cell membrane to be fluid in nature?

The fluid characteristic of the cell membrane allows greater flexibility to the cell than it would if the membrane were rigid. It also allows the motion of membrane components, required for some types of membrane transport.

Transcribe and translate the following DNA sequence (non-template strand): 5'-ATGGCCGGTTATTAAGCA-3'

The mRNA would be: 5′-AUGGCCGGUUAUUAAGCA-3′. The protein would be: MAGY. Even though there are six codons, the fifth codon corresponds to a stop, so the sixth codon would not be translated.

Why is it important that there are different types of proteins in plasma membranes for the transport of materials into and out of a cell?

The proteins allow a cell to select what compound will be transported, meeting the needs of the cell and not bringing in anything else.

Provide a brief summary of the Sanger sequencing method.

The template DNA strand is mixed with a DNA polymerase, a primer, the 4 deoxynucleotides, and a limiting concentration of 4 dideoxynucleotides. DNA polymerase synthesizes a strand complementary to the template. Incorporation of ddNTPs at different locations results in DNA fragments that have terminated at every possible base in the template. These fragments are separated by gel electrophoresis and visualized by a laser detector to determine the sequence of bases.

G protein linked receptors use a peripheral protein to activate an effector such as an enzyme

True

What characteristics make yeasts a good model for learning about signaling in humans?

Yeasts are eukaryotes and have many of the same systems that humans do; however, they are single-celled, so they are easy to grow, grow rapidly, have a short generation time, and are much simpler than humans.

A diploid, multicellular life-cycle stage that gives rise to haploid cells by meiosis is called a ________. a. sporophyte b. gametophyte c. spore d. gamete

a

If xenon has an atomic number of 54 and a mass number of 108, how many neutrons does it have? a. 54 b. 27 c. 100 d. 108

a

The ________ is the basic unit of life. a. organism b. cell c. tissue d. organ

b

A nucleotide of DNA may contain ________. a. ribose, uracil, and a phosphate group b. deoxyribose, uracil, and a phosphate group c. deoxyribose, thymine, and a phosphate group d. ribose, thymine, and a phosphate group

c

Beta-oxidation is ________. a. the breakdown of sugars b. the assembly of sugars c. the breakdown of fatty acids d. the removal of amino groups from amino acids

c

Cholesterol is an integral part of plasma membranes. Based on its structure, where is it found in the membrane? a. on the extracellular surface b. embedded with the phospholipid heads c. within the tail bilayer d. attached to the intracellular surface

c

During proofreading, which of the following enzymes reads the DNA? a. primase b. topoisomerase c. DNA pol d. helicase

c

How does the double helix structure of DNA support its role in encoding the genome? a. the sugar-phosphate backbone provides a template for DNA replication. b. tRNA pairing with the template strand creates proteins encoded by the genome. c. complementary base pairing creates a very stable structure. d. complementary base pairing allows for easy editing of both strands of DNA.

c

An example of a monosaccharide is ________. a. fructose b. glucose c. galactose d. all of the above

d

The diagram below shows a replication bubble within a DNA strand. A primer for DNA replication is 5'-GUACGUUG-3". There are multiple start/stop sites marked by letters and colors. Each letter corresponds to a color and is marked with a start-stop site and an arrow showing a direction in which replication would proceed. Which of the following correctly represents where the primer would anneal in the replication bubble and the direction of replication it would follow: A. A B. B C. C D. D E. E F. F G. G H. H

A. CATGCAA

A DNA sequence is several thousands of nucleotides distant from the promoter. When this sequence is bound by a protein, transcription rates increase greatly. This sequence is most likely a(n): A. TATA box. B. enhancer. C. operon. D. promoter. E. consensus sequence.

B

Why are biological macromolecules considered organic?

Biological macromolecules are considered organic because they contain a carbon ring or chain form.

How do cilia and flagella differ?

Cilia and flagella are alike in that they are made up of microtubules. Cilia are short, hair-like structures that exist in large numbers and usually cover the entire surface of the plasma membrane. Flagella, in contrast, are long, hair-like structures; when flagella are present, a cell has just one or two.

What are the structural differences between RNA and DNA?

DNA has a double-helix structure. The sugar and the phosphate are on the outside of the helix and the nitrogenous bases are in the interior. The monomers of DNA are nucleotides containing deoxyribose, one of the four nitrogenous bases (A, T, G and C), and a phosphate group. RNA is usually single-stranded and is made of ribonucleotides that are linked by phosphodiester linkages. A ribonucleotide contains ribose (the pentose sugar), one of the four nitrogenous bases (A, U, G, and C), and the phosphate group.

The presence of a membrane-enclosed nucleus is a characteristic of ________. A. bacteria B. animal cells C. eukaryotic cells D. prokaryotic cells E. both animal cells and eukaryotes are correct

E

Name two different cellular functions that require energy that parallel human energy-requiring functions.

Energy is required for cellular motion, through beating of cilia or flagella, as well as human motion, produced by muscle contraction. Cells also need energy to perform digestion, as humans require energy to digest food.

What are the differences between internal receptors and cell-surface receptors?

Internal receptors are located inside the cell, and their ligands enter the cell to bind the receptor. The complex formed by the internal receptor and the ligand then enters the nucleus and directly affects protein production by binding to the chromosomal DNA and initiating the making of mRNA that codes for proteins. Cell-surface receptors, however, are embedded in the plasma membrane, and their ligands do not enter the cell. Binding of the ligand to the cell-surface receptor initiates a cell signaling cascade and does not directly influence the making of proteins; however, it may involve the activation of intracellular proteins.

How does the role of meiosis in gamete production differ between organisms with a diploid-dominant lifecycle and organisms with an alternation of generations life cycle?

Organisms with a diploid-dominant life cycle make haploid gametes by meiosis, while all their somatic cells are diploid. Organisms with an alternation of generations life cycle make gametes during their haploid life stage, so the chromosome number does not need to be reduced, and meiosis is not involved.

How do the roles of ubiquinone and cytochrome c differ from the roles of the other components of the electron transport chain?

Q and cytochrome c are transport molecules. Their function does not result directly in ATP synthesis in that they are not pumps. Moreover, Q is the only component of the electron transport chain that is not a protein. Ubiquinone and cytochrome c are small, mobile electron carriers, whereas the other components of the electron transport chain are large complexes anchored in the inner mitochondrial membrane.

Mendel performs a cross using a true-breeding pea plant with round, yellow seeds and a true-breeding pea plant with green, wrinkled seeds. What is the probability that offspring will have green, round seeds? Calculate the probability for the F1 and F2 generations.

Since we are calculating the probability of two independent events occurring simultaneously, we use the product rule. F1 generation: Since green seed color is recessive, there is a 0% probability that any plants in the F1 generation will have green, round seeds. F2 generation: The probability of growing an F2 generation plant with green seeds is ¼, while the probability of growing an F2 generation plant with round seeds is ¾. We can use the product rule to then calculate the probability of a plant with green, round seeds: ¼ × ¾ = 3/16

How do the linear chromosomes in eukaryotes ensure that its ends are replicated completely?

Telomerase has an inbuilt RNA template that extends the 3′ end, so primer is synthesized and extended. Thus, the ends are protected.

How are the effects of paracrine signaling limited to an area near the signaling cells?

The secreted ligands are quickly removed by degradation or reabsorption into the cell so that they cannot travel far.

An adult with a history of tanning has his genome sequenced. The beginning of a protein-coding region of his DNA reads ATGGGGATATGGCAT. If the protein-coding region of a healthy adult reads ATGGGGATATGAGCAT, identify the site and type of mutation.

This is a frameshift mutation with a deletion of an "A" in the 12th position of the coding region. · Patient: ATGGGGATATGGCAT · Normal: ATGGGGATATGAGCAT

The hydrolysis of one ATP molecule releases 7.3 kcal/mol of energy (∆G = −7.3 kcal/mol of energy). If it takes 2.1 kcal/mol of energy to move one Na+ across the membrane (∆G = +2.1 kcal/mol of energy), how many sodium ions could be moved by the hydrolysis of one ATP molecule?

Three sodium ions could be moved by the hydrolysis of one ATP molecule. The ∆G of the coupled reaction must be negative. Movement of three sodium ions across the membrane will take 6.3 kcal of energy (2.1 kcal × 3 Na+ ions = 6.3 kcal). Hydrolysis of ATP provides 7.3 kcal of energy, more than enough to power this reaction. Movement of four sodium ions across the membrane, however, would require 8.4 kcal of energy, more than one ATP molecule can provide.

Cytoplasmic receptors bind non-polar molecules

True

Energy can neither be created nor destroyed

True

A major connection for sugars in glycolysis is ________. a. glucose-6-phosphate b. fructose-1,6-bisphosphate c. dihydroxyacetone phosphate d. phosphoenolpyruvate

a

Cohesion proteins binding the sister chromatids break down

anaphase

The -10 and -35 regions of prokaryotic promoters are called consensus sequences because ________. a. they are identical in all bacterial species b. they are similar in all bacterial species c. they exist in all organisms d. they have the same function in all organisms

b

Glycolysis can be considered as consisting of two parts. These parts are the ______________ and the ______________ phase.

preparatory; production

In the context of cell biology, what do we mean by form follows function? What are at least two examples of this concept?

"Form follows function" refers to the idea that the function of a body part dictates the form of that body part. As an example, organisms like birds or fish that fly or swim quickly through the air or water have streamlined bodies that reduce drag. At the level of the cell, in tissues involved in secretory functions, such as the salivary glands, the cells have abundant Golgi.

A cell develops a mutation in its potassium channels that prevents the ions from leaving the cell. If the cell's aquaporins are still active, what will happen to the cell? A. water will not leave the cell; increasing cell volume B. water will leave the cell; decreasing cell volume C. potassium will leave the cell through aquaporins D. potassium will enter the cell through aquaporins E. both a and c are correct

A

A nucleotide of DNA may contain ________. A. deoxyribose, thymine, and a phosphate group B. ribose, uracil, and a phosphate group C. deoxyribose, amino acids, and a phosphate group D. deoxyribose, uracil, and a phosphate group E. ribose, thymine, and a phosphate group

A

Based on the structure of the phospholipid bilayer, which of the following would be found embedded within the center / tail bilayer of the membrane? A. cholesterol B. glycoprotein C. disaccharide D. protein E. ion channel

A

Cellulose and starch are examples of: A. both isomers and polysaccharide are correct B. lipids C. isomers D. monosaccharides E. polysaccharides

A

Chromosomes "decondense" into diffuse chromatin: A. at the end of telophase. B. at the beginning of prophase. C. at the end of interphase. D. at the end of metaphase. E. only in dying cells.

A

Dinitrophenol (DNP) is an "uncoupler" that makes the inner mitochondrial membrane "leaky" to protons so that the electron transport chain can no longer form a proton gradient, which stops ATP synthase and the production of ATP. Which of the following best describes why hyperthermia (i.e. overheating) is one of the side effects of DNP poisoning? A. The second law of thermodynamics explains why hypethermia occurs B. Gibbs free energy calculations explains why hyperthermia occurs C. All of these are correct D. The law of enthalpy explains why hyperthermia occurs E. The relationship between potential and kinetic energy explains why hyperthermia occurs

A

During mitosis and meiosis the chromatin compacts. Which of the following processes takes place more easily because of this compaction? A. The orderly distribution of genetic material to two new nuclei B. DNA replication C. Exposing of the genetic information on the DNA D. The unwinding of DNA from the histones E. The disappearance of the nuclear membrane

A

Each carbon molecule can bond with as many as________ other atom(s) or molecule(s). Answers: A. 4 B. 0 C. 6 D. 8 E. 1

A

Electrochemical gradients in neurons are established using sodium-potassium pumps. Which of the following transport system is used to establish the gradient? A. Primary active transport B. Passive transport C. Facilitated passive transport D. Secondary active transport E. None of these

A

Glucose from digested food enters intestinal epithelial cells by a form of transport that is different from many other body cells. Which of the following best describes why intestinal cells use a different form of transport? A. Intestinal cells can use active glucose transport because the glucose concentration gradient may not always be favorable for facilitated diffusion to occur. B. Glucose is never transported using active transport. It is always transported into intestinal cells using facilitated diffusion. C. Intestinal cells use active glucose transport because glucose can never enter a cell through facilitated diffusion. D. Intestinal cells use active glucose transport because glucose is a large molecule and facilitated diffusion uses channel proteins that select out large macromolecules. E. Intestinal cells use active glucose transport because glucose is strongly electronegative and facilitated diffusion uses channel proteins that select out large macromolecules.

A

If a double-stranded DNA molecule contains 30 percent T, it must contain _______ percent G. A. 20 B. 30 C. 40 D. 50 E. 60

A

If you accidentally ingest methanol (CH3OH) instead of ethanol, the enzyme ADH produces toxic FORMALDEHYDE, which is in turn converted by the next enzyme in the pathway, aldehyde dehydrogenase (ALDH), into formic acid. Formic acid is especially toxic and strongly affects the optic nerve, so methanol poisoning can cause blindness. The treatment for methanol is, perhaps surprisingly, to give the patient ethanol (regular grain alcohol). The ethanol inhibits the ADH reaction that is converting the methanol to formaldehyde and formic acid. Given the structures of methanol and ethanol below, which of the following explain why ethanol treatment works to stop the toxic effects of the methanol poisoning. A. Ethanol is a competitive inhibitor of methanol. It competes with methanol to bind to the active site of ADH and thereby stops the conversion of methanol into formaldehyde. B. Ethanol is a non-competitive inhibitor of methanol. It binds to a separate site on a substrate to inhibit the activity of ADH. C. Ethanol breaks the bonds between carbon and hydroxyl group bonds and adds this to the methanol. This causes methanol toxicity to be eliminated. D. Methanol and ethanol form a bond together through the hydroxyl group. This is an endergonic reaction that causes the formation of isopropanol , which is not toxic. E. Methanol is metabolized in an endergonic reaction in the presence of ethanol F. Both D and E are correct

A

If you place red blood cell into a sucrose solution of 0.3 M and you determine this solution is an isotonic concentration, what would be the isotonic concentration if you were to put MgCl2 into the extracellular fluid instead? A. 0.1 M B. 0.3 M C. 0.6 M D. 0.9 M E. 0.01 M

A

Suppose that two different groups of schmoos (an imaginary animal) live in geographically separated locations and rarely interbreed. On one occasion, a big-footed white-haired schmoo does mate with a small-footed brown-haired schmoo. Six offspring result: two big-footed schmoos with light brown hair and four small-footed schmoos with light brown hair. Which of the following statements about the inheritance of hair color in schmoos is most likely correct? A. There is incomplete dominance. B. Brown is dominant to white. C. White is dominant to brown. D. White and brown are codominant. E. This cannot be answered without more information.

A

The cells in your intestinal walls frequently divide. Therefore, these cells are likely to: A. have high levels of telomerase. B. have very low levels of telomerase. C. undergo the polymerase chain reaction. D. lack DNA polymerase. E. lack Okazaki fragments.

A

The genotype of an organism that expresses a dominant trait can be determined by: A. crossing the organism with a homozygous recessive organism. B. crossing the organism with a heterozygous dominant organism. C. crossing the organism with a homozygous dominant organism. D. observing the phenotype of the progeny from any cross. E. observing the genotype of the progeny from any cross.

A

The protein channels in the membrane of neurons are highly selective. These channels only allow specific ions to pass through the neuron membrane. Which of the following accurately describes how these channels may select for specific ions? A. The charge and polarity of the R groups that line the pore of the channel can attract and repel various ions. B. The channel protein is selective because the channel protein contains a lipid gate mechanism that is removed by specific charged ions. C. The channel pore selects ions based on size D. All of these accurately describe how protein channels may select for specific ions E. The channel protein is selective because of the alpha helix configuration of the proteins that make up the channel.

A

The trp operon: A. codes for proteins needed for tryptophan synthesis. B. codes for proteins needed to metabolize tryptophan. C. is activated by the presence of tryptophan. D. is inducible by tryptophan. E. is inducible by lactose (or allolactose).

A

This molecule has three phosphate groups; named for the order in which they align away from a ribose sugar. A. ATP B. NAD C. ADP D. NADH E. NADPH

A

Which characteristic of a phospholipid contributes to the fluidity of the membrane? A. the fatty acid tails B. hydrogen bonds in water C. proteins in the membrane D. phosphate group E. all of these contribute to the fluidity of plasma membrane

A

Which of the following best describes why closing stomata limits photosynthesis? A. the plant cannot acquire new carbon dioxide molecules, which limits the light-independent reactions which will only occur until CO2 stores are depleted. B. the plant cannot release oxygen which limits photosynthesis C. the plant cannot acquire water, which is needed by photosystem I in the chloroplasts D. the plant cannot acquire water, which is needed by photosystem II in the chloroplasts. E. the plant cannot acquire oxygen, which is needed during the Calvin Cycle to make glucose.

A

Which of the following correctly describes endosymbiosis? A. A hypothesis that mitochondria and chloroplast were once prokaryotic organisms that had a mutually beneficial relationship with a eukaryote. Over evolutionary time, the prokaryote eventually became an organelle within the eukaryote. B. A hypothesis that the nucleus was once a prokaryotic organism that had a parasitic relationship with a eukaryote. Over evolutionary time, the prokaryote eventually became the nucleus within the eukaryote. C. A hypothesis that eukaryotic organisms evolved from prokaryotic organisms via symbiotic relationships. D. Both A and C are correct E. All of these are correct

A

Which of the following correctly describes what will happen in this cell? 0.15 M NaCl (inside the cell); 0.15 M NaCl, 1.5 M CO2 (both outside the cell) A. This solution is isotonic. The volume of the cell will be unchanged. The CO2 will cross the cell membrane until the partial pressure of CO2 is evenly distributed on both sides of the membrane. B. This solution is hypertonic. The volume of the cell will decrease. The CO2 adds to the tonicity of the solution, which will cause water to leave the cell and shrivel. C. This solution is hypotonic. The volume of the cell will increase The CO2 binds the disassociated chlorine molecules from the NaCl and this decreases the tonicity of the solution. D. Water will enter the cell until it ruptures. E. C and D are both correct

A

Which of the following correctly describes why an individual with a mutation that prevents recombination during meiosis might be (or might not be) considered less fit than other members of its species? A. The chromosomes of the individual cannot cross over during meiosis. This limits the genetic diversity of the individual's gametes because independent assortment is the only remaining mechanism to promote genetic diversity in the gametes. An individual who cannot produce diverse offspring is considered less fit than individuals who do produce diverse offspring. B. The chromosomes of the individual cannot independently assort during meiosis. This limits the genetic diversity of the individual's gametes because crossing over is the only remaining mechanism to promote genetic diversity in the gametes. An individual who cannot produce diverse offspring is not considered less fit than individuals who do produce diverse offspring as this will have no effect on reproduction and therefore fitness of the individual. C. The chromosomes of the individual cannot independently assort during meiosis. This limits the genetic diversity of the individual's gametes because independent assortment is the only way to promote genetic diversity in the gametes. An individual who cannot produce diverse offspring is considered less fit than individuals who do produce diverse offspring. D. The chromosomes of the individual cannot cross over during meiosis. This limits the genetic diversity of the individual's gametes because independent assortment is the only remaining mechanism to promote genetic diversity in the gametes. An individual who cannot produce diverse offspring is not considered less fit than individuals who do produce diverse offspring as this will have no effect on reproduction and therefore fitness of the individual. E. The chromosomes of the individual cannot independently assort during meiosis. This limits the genetic diversity of the individual's gametes because crossing over is the only remaining mechanism to promote genetic diversity in the gametes. An individual who cannot produce diverse offspring is considered less fit than individuals who do produce diverse offspring.

A

Which of the following correctly identifies this structure and describes why the structure's appropriately name? A. This is an amino acid. The carboxyl group in section 3 is why it is considered an acid and the group in section 1 is considered an amino group. B. This is a nucleotide. The group in section 3 is an amino and the group in section 1 is the carbon backbone. C. This is a nucleotide. The group in section 1 is an amino group and the group in section 2 is the carbon backbone. D. This is an amino acid. The carboxyl group in section 3 is why it is considered an acid and the group in section 2 is considered an amino group. E. This is an amino acid. The carboxyl group in section 1 is why it is considered an acid and the group in section 3 is an amino group.

A

Which of the following describes why the second phase of cellular respiration is a cyclic process? A. The citric acid cycle is completed by a 4 carbon structure that accepts an acetyl group from the substrate that starts the cycle; this forms the 6 carbon structure in the first step of the process. B. The citric acid cycle is completed by a 6 carbon structure accepts an acetyl group from the substrate that starts the cycle; this forms the 8 carbon structure in the first step of the process. C. The phosphate groups from the hydrolysis of ATP during the Calvin cycle are added to the 3 carbon structures needed to start the process. D. ADP from the Calvin cycle are cycled back into the first steps in the process where they are needed. E. Electron transport chain is completed by a 4 carbon structure accepts an acetyl group from the substrate that starts the cycle; this forms the 6 carbon structure in the first step.

A

Which of the following sequences correctly lists in order the steps involved in the incorporation of a proteinaceous molecule within a cell? A. synthesis and modification of the protein on the ribosome and endoplasmic reticulum; followed by addition of sugar groups in the Golgi; then distribution in the cell via the vesicle. B. synthesis and modification of the protein on the ribosome and endoplasmic reticulum followed by addition of sugar groups in the lysosomes then distribution in the cell via microtubules. C. synthesis and modification of the protein in the nucleus and endoplasmic reticulum followed by addition of sugar groups in the Golgi then distribution in the cell via the microfilaments. D. synthesis of the protein on the ribosome; followed my modification in the lysosomes; followed by addition of sugar groups in the Golgi then distribution in the cell via the vesicle. E. synthesis of the protein in the nucleus; followed by protein modification in the endoplasmic reticulum; followed by addition of sugar groups in the Golgi; then distribution in the cell via the vesicle.

A

A mistake during meiosis I that results in chromosomes not fully separating from each other. Homologous chromosomes or sister chromatids fail to separate resulting in an abnormal additional chromosome number.

A and B

Explain what happens if even one amino acid is substituted for another in a polypeptide chain. Provide specific example.

A change in gene sequence can lead to a different amino acid being added to a polypeptide chain instead of the normal one. This causes a change in protein structure and function. For example, in sickle cell anemia, the hemoglobin β chain has a single amino acid substitution—the amino acid glutamic acid in position six is substituted by valine. Because of this change, hemoglobin molecules form aggregates, and the disc-shaped red blood cells assume a crescent shape, which results in serious health problems.

In rats, an enzyme that is 192 amino acids long is coded for by a gene that is 1440 bp long. In addition to the fact that it takes 3 nucleotides to code for one amino acid, there are two different sources for the discrepancy between the DNA and the amino acids. Identify and explain these sources of discrepancy.

A combination of any of the following: non coding regions, promoter regions, enhancer regions, repressor regions, introns, poly A tail. Must include more than one of these options

Look at each of the processes shown (Figure 6.8 p. 182 in textbook), and decide if it is endergonic or exergonic. In each case, does enthalpy increase or decrease, and does entropy increase or decrease?

A compost pile decomposing is an exergonic process; enthalpy increases (energy is released) and entropy increases (large molecules are broken down into smaller ones). A baby developing from a fertilized egg is an endergonic process; enthalpy decreases (energy is absorbed) and entropy decreases. Sand art being destroyed is an exergonic process; there is no change in enthalpy, but entropy increases. A ball rolling downhill is an exergonic process; enthalpy decreases (energy is released), but there is no change in entropy.

If the pH outside the cell decreases, would you expect the amount of amino acids transported into the cell to increase or decrease?

A decrease in pH means an increase in positively charged H+ ions, and an increase in the electrical gradient across the membrane. The transport of amino acids into the cell will increase.

In your everyday life, you have probably noticed that certain instruments are ideal for certain situations. For example, you would use a spoon rather than a fork to eat soup because a spoon is shaped for scooping, while soup would slip between the tines of a fork. The use of ideal instruments also applies in science. In what situation(s) would the use of a light microscope be ideal, and why?

A light microscope would be ideal when viewing a small living organism, especially when the cell has been stained to reveal details.

Describe how microfilaments and microtubules are involved in the phagocytosis and destruction of a pathogen by a macrophage.

A macrophage engulfs a pathogen by rearranging its actin microfilaments to bend the plasma membrane around the pathogen. Once the pathogen is sealed in an endosome inside the macrophage, the vesicle is walked along microtubules until it combines with a lysosome to digest the pathogen.

A sequence of 120 base pairs of a DNA molecule is shown below. While it is possible for transcription to occur on both strands it is very rare and therefore the VAST majority (i.e. most) of transcription occurs on one strand. Assume that transcription begins at the underlined A/T (top strand/bottom strand) at base pair 41 on the commonly used strand. This is labeled site (a) in the diagram below. Transcription proceeds to the right. What are the first 12 nucleotides of the resulting mRNA?

AAACAGCUAUGG

A fragment of bacterial DNA reads: 3'-TACCTATAATCTCAATTGATAGAAGCACTCTAC-5'. Assuming that this fragment is the template strand, what is the sequence of mRNA that would be transcribed? (Hint: Be sure to identify the initiation site.)

ACUAUCUUCGUGAGAUG By examining the DNA sequence, we can see that there is a -10 consensus sequence near the 3' end of the fragment. If we then count downstream, the +1 initiation site is the T immediately following the sequence AAT. This means the DNA fragment that will serve as the template for transcription has the sequence TGATAGAAGCACTCTAC. The mRNA made from this template will have complimentary base pairing with uracil (U) instead of thymine (T). This gives us ACUAUCUUCGUGAGAUG as the transcribed mRNA sequence.

Why is it beneficial for cells to use ATP rather than energy directly from the bonds of carbohydrates? What are the greatest drawbacks to harnessing energy directly from the bonds of several different compounds?

ATP provides the cell with a way to handle energy in an efficient manner. The molecule can be charged, stored, and used as needed. Moreover, the energy from hydrolyzing ATP is delivered as a consistent amount. Harvesting energy from the bonds of several different compounds would result in energy deliveries of different quantities.

What kind of sugars are these, aldose or ketose?

According to the figure shown, the glucose and galactose has an aldehyde group which forms a double bond with the carbon and the forming a single bond with the carbon. Therefore, they are aldose sugars. However, in fructose sugars, we see the replacement of hydrogen by the carbonyl group. Therefore, fructose is a ketose sugar.

A mutation occurs, and cytosine is replaced with adenine. What impact do you think this will have on the DNA structure?

Adenine is larger than cytosine and will not be able to base pair properly with the guanine on the opposing strand. This will cause the DNA to bulge. DNA repair enzymes may recognize the bulge and replace the incorrect nucleotide.

Prokaryotes have a single circular chromosome while eukaryotes have linear chromosomes. Describe one advantage and one disadvantage to the eukaryotic genome packaging compared to the prokaryotes.

Advantage: The linear arrangement of the eukaryotic chromosome allows more DNA to be packed by tightly winding it around histones. More genetic material means that the organism can encode more information into a single cell. This eventually allowed some eukaryotes to develop into multicellular organisms with cell specialization. Disadvantage: Maintaining more genetic material requires more energy, and introduces the possibility for more errors (more complexity).

Dinitrophenol (DNP) is an "uncoupler" that makes the inner mitochondrial membrane "leaky" to protons. It was used until 1938 as a weight-loss drug. What effect would you expect DNP to have on the change in pH across the inner mitochondrial membrane? Why do you think this might be an effective weight-loss drug?

After DNP poisoning, the electron transport chain can no longer form a proton gradient, and ATP synthase can no longer make ATP. DNP is an effective diet drug because it uncouples ATP synthesis; in other words, after taking it, a person obtains less energy out of the food he or she eats. Interestingly, one of the worst side effects of this drug is hyperthermia, or overheating of the body. Since ATP cannot be formed, the energy from electron transport is lost as heat.

Cyanide inhibits cytochrome c oxidase, a component of the electron transport chain. If cyanide poisoning occurs, would you expect the pH of the intermembrane space to increase or decrease? What effect would cyanide have on ATP synthesis?

After cyanide poisoning, the electron transport chain can no longer pump electrons into the intermembrane space. The pH of the intermembrane space would increase, the pH gradient would decrease, and ATP synthesis would stop.

Because they lose their mitochondria during development, red blood cells cannot perform aerobic respiration; however, they do perform glycolysis in the cytoplasm. Why do all cells need an energy source, and what would happen if glycolysis were blocked in a red blood cell?

All cells must consume energy to carry out basic functions, such as pumping ions across membranes. A red blood cell would lose its membrane potential if glycolysis were blocked, and it would eventually die.

In a comparison of the stages of meiosis to the stages of mitosis, which stages are unique to meiosis and which stages have the same events in both meiosis and mitosis?

All of the stages of meiosis I, except possibly telophase I, are unique because homologous chromosomes are separated, not sister chromatids. In some species, the chromosomes do not decondense and the nuclear envelopes do not form in telophase I. All of the stages of meiosis II have the same events as the stages of mitosis, with the possible exception of prophase II. In some species, the chromosomes are still condensed and there is no nuclear envelope. Other than this, all processes are the same.

Compare and contrast the boundaries that plant, animal, and bacteria cells use to separate themselves from their surrounding environment.

All three cell types have a plasma membrane that borders the cytoplasm on its interior side. In animal cells, the exterior side of the plasma membrane is in contact with the extracellular environment. However, in plant and bacteria cells, a cell wall surrounds the outside of the plasma membrane. In plants, the cell wall is made of cellulose, while in bacteria the cell wall is made of peptidoglycan. Gram-negative bacteria also have an additional capsule made of lipopolysaccharides that surrounds their cell wall.

Amino acids have the generic structure seen below, where R represents different carbon-based side chains. Describe how the structure of amino acids allows them to be linked into long peptide chains to form proteins.

Amino acids can be linked into long chains through condensation reactions. One of the hydrogen atoms bonded to the nitrogen atom of an amino acid reacts with the -f group attached to the terminal carbon on another amino acid. Since both ends of the molecule can participate in condensation reactions, peptide bonds can be made in both directions to create a long amino acid chain.

Imagine a sealed terrarium containing a plant and a beetle. How does each organism provide resources for the other? Could each organism survive if it was the only living thing in the terrarium? Why or why not?

An energy cycle between a plant and a beetle would be as follows: A) Plant consumes carbon dioxide and releases oxygen as a byproduct of photosynthesis B) Beetle consumes oxygen and releases carbon dioxide to create chemical energy during aerobic respiration C) Plant takes up carbon dioxide from the air D) Repeat cycle The plant would also provide a carbon-based food source for the beetle. 1) The beetle is a heterotroph, and would not survive without the plant because it would deplete all the oxygen within the terrarium. 2) The plant is an autotroph and could survive without the beetle, but it would be unlikely to grow. Through photosynthesis, the plant can make and store its own energy in carbon-based molecules, and produce oxygen. The oxygen can then be used to power aerobic respiration in the plant, which releases carbon dioxide. However, since the plant essentially continues to reuse its own resources cycling between carbon- and oxygen-consuming pathways, its growth would be limited.

DNA replication is bidirectional and discontinuous; explain your understanding of those concepts.

At an origin of replication, two replication forks are formed that are extended in two directions. On the lagging strand, Okazaki fragments are formed in a discontinuous manner.

Explain the events taking place at the replication fork. If the gene for helicase is mutated, what part of replication will be affected?

At the replication fork, the events taking place are helicase action, binding of single-strand binding proteins, primer synthesis, and synthesis of new strands. If there is a mutated helicase gene, the replication fork will not be extended.

A hydrophobic chemical signal will use which type of receptor to initiate which of the following cell signaling pathways? A. A cell membrane receptor that initiates second messenger systems B. An intracellular receptor that initiates a pathway that ends in modified gene transcription. C. A G-protein linked receptor that initiates transcription factors D. A channel protein receptor that initiates ion current into the cell. E. A tyrosine kinase receptors that initiates dimerization of peripheral proteins

B

A scientist observes that a cell has an RNA polymerase deficiency that prevents it from making proteins. Which of the following describes observations that would support the conclusion that there is a defect in RNA polymerase I activity and not a defect with the any of the other polymerases? (visit your text for a complete list of all that these enzymes do) A. Observations eliminating RNA polymerase II as the defect could include: transcription of mRNAs in the nucleus The observations eliminating RNA polymerase III could include: Isolation of small nuclear RNAs from the cell B. A and C would support the conclusion that there is a defect in RNA polymerase I activity C. Observations eliminating RNA polymerase II as the defect could include: presence of processed mRNAs in the cytoplasm. The observations eliminating RNA polymerase III could include: isolation of microRNAs from the cell, presence of tRNAs in the cytoplasm. D. Observations eliminating RNA polymerase III as the defect could include: transcription of mRNAs in the nucleus and the presence of processed mRNAs in the cytoplasm. The observations eliminating RNA polymerase II could include: Isolation of small nuclear RNAs from the cell, isolation of microRNAs from the cell, transcription of 5S rRNA in the nucleus, presence of tRNAs in the cytoplasm.

B

According to the following descriptions, which of the genes (A-E) is most likely regulated at the posttranscriptional level? A. Gene A, which shows a strong positive correlation between the amount of mRNA produced and the amount of protein produced B. Gene B, which shows no clear correlation between the amount of mRNA produced and the amount of protein produced C. Gene C, which lies near the heterochromatin D. Gene D, which has more than a typical number of CpG islands E. Gene E, which has fewer than a typical number of CpG islands

B

Consider all the functions of a plasma membrane. What membrane functions would have been important to start the evolution of multi-cellular organisms for single celled organisms? A. Second messenger systems B. Cell adhesion and intracellular joining C. enzymatic activity D. signal transduction E. Transport

B

During DNA replication: A. one template strand must be degraded to allow the other strand to be copied. B. the template strands must separate so that both can be copied. C. the template strands come back together after the passage of the replication fork. D. origins of replication always give rise to single replication forks. E. two replication forks diverge from each origin but one always lags behind the other.

B

If a cell in G2 has 1.6 picograms (pg) of DNA, it had _______ of DNA in G1. A. 0.4 pg B. 0.8 pg C. 1.6 pg D. 3.2 pg E. 1.6 pg if it is a plant cell and 3.2 pg if it is an animal cell

B

Imagine if there were 200 commonly occurring amino acids instead of 20. Given what you know about the genetic code, what would be the shortest possible codon length? A. For 200 commonly occurring amino acids, codons consisting of three types of nucleotides would have to be at least three nucleotides long, because 44 = 256. There would be much less degeneracy in this case. B. For 200 commonly occurring amino acids, codons consisting of four types of nucleotides would have to be at least four nucleotides long, because 44 = 256. There would be much less degeneracy in this case. C. For 200 commonly occurring amino acids, codons consisting of five types of nucleotides would have to be at least five nucleotides long, because 44 = 256. There would be much less degeneracy in this case. D. None of these fit what I know about the genetic code

B

In humans, a widow's peak is caused by a dominant allele (W), and a continuous hairline is caused by a recessive allele (w). Short fingers are caused by a dominant allele (S), and long fingers are caused by a recessive allele (s). Suppose a woman with a continuous hairline and short fingers and a man with a widow's peak and long fingers have three children. One child has a widow's peak and short fingers, one has a widow's peak and long fingers, and one has a continuous hairline and long fingers. What are the genotypes of the parents? A. Female wwSS; male WWss B. Female wwSs; male Wwss C. Female wwSs; male WWss D. Female WwSs; male WwSs E. None of the above

B

Ionotropic receptors are also an example of which of the following? A. Primary active transport B. Facilitated passive transport C. None of these D. Secondary active transport E. Passive transport

B

The enzyme DNA ligase is required continuously during DNA replication in order for: A. fragments of the leading strand to be joined together. B. fragments of the lagging strand to be joined together. C. the parental strands to be joined back together. D. 3ʹ-deoxynucleoside triphosphates to be converted to 5ʹ-deoxynucleoside triphosphates. E. the complex of proteins that work together at the replication fork to remain intact.

B

The first repair of mistakes during DNA replication is made by: A. the mismatch repair system. B. DNA polymerase. C. excision repair. D. SOS repair. E. post-replication repair.

B

The monomers that make up proteins are called ________. A. cytosine, thymine, adenine or guanine B. amino acids C. saccharides D. nucleotides E. fatty acids

B

The total DNA content of each daughter cell is reduced during meiosis because: A. chromosomes do not replicate during the interphase preceding meiosis I. B. chromosomes do not replicate between meiosis I and II. C. half of the chromosomes from each gamete are lost during fertilization. D. sister chromatids separate during anaphase of meiosis I. E. chromosome arms are lost during crossing over.

B

Which of the following accepts electrons from a substrate during an enzymatic reaction and which of the following donates electrons? A. Oxygen accepts electrons whereas O2 donates electrons B. NAD+ accepts electrons whereas NADH donates electrons. C. NADH accepts electrons whereas NAD+ donates electrons. D. Enzymes accept electrons whereas coenzymes donate electrons E. FADH accepts electrons whereas FAD+ donates electrons

B

Which of the following are found only in plant cells? A. vacuoles B. all of these are only found in plant cells C. cell wall made from cellulose D. chloroplast E. plasmodesmata

B

Which of the following best describes why yeasts are good organism for learning about human cell functions? A. Because yeasts are prokaryotes which helps us to understand how human cell signaling pathways are altered after bacterial infection. B. Because yeasts are eukaryotes and have the same signaling pathway systems as humans but also have a short generation time which makes them easier to grow and simpler to study in a lab than human cells. C. Because yeasts are prokaryotes and have the same signaling pathway systems as humans but also have a short generation time which makes them easier to grow and simpler to study in a lab than human cells. D. Because yeasts are not eukaryotic or prokaryotic and the unique signaling pathways in yeast cells provide information about the evolution of human signaling pathways. E. Because yeasts are eukaryotes which helps us to understand how human cell signaling pathways are altered after bacterial infection.

B

Which of the following correctly describes the reaction occurring below? A. hydrolysis of disaccharide B. hydrolysis of an ester C. hydrolysis of a protein D. condensation reaction of a protein E. condensation reaction of a disaccharide

B

Which of the following correctly explains how eukaryotic chromosomes are thousands of times longer than a typical cell and yet still fit inside a eukaryotic nucleus? A. The DNA double helix is wrapped around microtubles to form structures called spindle fibers. Nucleosomes and the linker DNA in between them are coiled into a 30-nm fiber. During cell division, chromatin is further condensed by packing proteins. B. The DNA double helix is wrapped around histone proteins to form structures called nucleosomes. Nucleosomes and the linker DNA in between them are coiled into a 30-nm fiber. During cell division, chromatin is further condensed by packing proteins. C. The DNA double helix is wrapped around histone which are comprised of nucleotides and ribose. Together, these form structures called nucleosomes. Nucleosomes coiled into a 30-nm fibers that condense during interphase and relax in cell division. D. Each strand of the DNA double helix is individually wrapped around histone proteins to form structures called nucleosomes. Nucleosomes have linker DNA that keeps the DNA double helix in tact. During cell division, chromatin is further condensed by packing proteins. E. During interphase, DNA double helix is wrapped around histone proteins to form structures called nucleosomes. Nucleosomes and the linker DNA in between them are coiled into a 30-nm fiber. During interphase, chromatin is further condensed by packing proteins and relaxes during cell division.

B

Which of the following describes a difference between proton gradients during cellular respiration and photosynthesis? A. There are no differences in proton gradients between cellular respiration and photosynthesis. In both cases, proton gradients regulate ATP synthase. B. Protons are pumped outside the mitochondria membrane in cellular respiration whereas in photosynthesis the protons are pumped to the inside of the chloroplast. C. Proton gradients are used as a form of potential energy in cellular respiration whereas in photosynthesis, protons are only added to NADP to form NADPH. D. Proton gradients do not occur in cellular respiration. They do occur in photosynthesis E. Proton gradients do not occur in photosynthesis. They only occur in respiration.

B

Which of the following is a correct statement regarding ribosomes? A. Ribosomes can be free floating in the cytoplasm or contained within the rough endoplasmic reticulum B. All of these statements are correct with regard to ribosomes C. Ribosomes contain both large and small subunits D. Ribosome synthesis is directed by the nucleolus E. Both prokaryotes and eukaryotes have ribosomes

B

Which of the following is not a functional group that can bond with carbon? A. all of these are functional groups that bond to carbon B. sodium C. phosphate D. hydroxyl E. carbonyl

B

Which of the following is not required for transcription? A. A DNA template B. A primer C. Ribonucleoside triphosphates D. RNA polymerase E. All of the above are required for transcription.

B

Which of the following processes will not stop if oxygen is depleted? A. Oxidative phosphorylation B. Glycolysis C. Electron transport chain D. Citric acid cycle E. Krebs cycle

B

Which of the following types of mutations involves two different nonhomologous chromosomes? A. Inversions B. Translocations C. Duplications D. Both a and b E. Both a and c

B

Which of the following will amplify the intracellular effects of a signal that has bound to a receptor? A. Ion channel receptors that allow the movement of sodium B. G proteins that initiate second messenger systems C. All of these amplify signals D. Proton pumps that move H+ to one side of the plasma membrane E. Secondary active transporters that shuttle ions into the cell

B

Which of the following would NOT terminate a signal transduction pathway? A. A decrease in receptor availability for active proteins within the pathway. B. All of these would terminate a signal transduction pathway. C. An increase in enzymes that inactive or oppose the activity of the proteins in the pathway would terminate activity in the pathway. D. The re-uptake or recycling of protein signals terminates activity within the pathway E. The breakdown of protein signals terminates activity within the pathway

B

Which part of the light-independent reactions would be affected if a cell could not produce the enzyme RuBisCO? A. This would inhibit the production of glyceraldyde 3 phosphates only B. This would inhibit the production of everything in the light-independent reaction. C. This would influence the production of ribulose bisphosphate only D. This would inhibit the production of ATP only E. This would inhibit the production of ATP and NADPH

B

Why does it take three turns of the Calvin cycle to produce G3P, the initial product of photosynthesis?

Because G3P has three carbon atoms, and each turn of the cycle takes in one carbon atom in the form of carbon dioxide.

Why is the third stage of the Calvin cycle called the regeneration stage?

Because RuBP, the molecule needed at the start of the cycle, is regenerated from G3P.

The gene for flower position in pea plants exists as axial or terminal alleles. Given that axial is dominant to terminal, list all of the possible F1 and F2 genotypes and phenotypes from a cross involving parents that are homozygous for each trait. Express genotypes with conventional genetic abbreviations.

Because axial is dominant, the gene would be designated as A. F1 would be all heterozygous Aa with axial phenotype. F2 would have possible genotypes of AA, Aa, and aa; these would correspond to axial, axial, and terminal phenotypes, respectively.

Why are carnivores, such as lions, dependent on photosynthesis to survive?

Because lions eat animals that eat plants.

What are the structural and functional similarities and differences between mitochondria and chloroplasts?

Both are similar in that they are enveloped in a double membrane, both have an intermembrane space, and both make ATP. Both mitochondria and chloroplasts have DNA, and mitochondria have inner folds called cristae and a matrix, while chloroplasts have chlorophyll and accessory pigments in the thylakoids that form stacks (grana) and a stroma.

A fragment of bacterial DNA reads: 3' -TACCTATAATCTCAATTGATAGAAGCACTCTAC- 5' Assuming that this fragment is the template strand, what is the sequence of mRNA that would be transcribed and which explanation correctly explains this? (Hint: Be sure to identify the initiation site.) A. TGATAGAAGCACTCTAC By examining initiation we can see that the DNA fragment that will serve as the template for transcription has the sequence ACUAUCUUCGUGAGAUG with uracil (U) instead of thymine (T). The mRNA made from this template will have complementary base pairing . B. ACTATCTTCGTGAGATG By examining initiation we can see that the DNA fragment that will serve as the template for transcription has the sequence TGATAGAAGCACTCTAC. The mRNA made from this template will have complementary base pairing . C. ACUAUCUUCGUGAGAUG By examining the DNA sequence, we can see that there is a -10 consensus sequence near the 3' end of the fragment. If we then count downstream, the +1 initiation site is the T immediately following the sequence AAT. This means the DNA fragment that will serve as the template for transcription has the sequence TGATAGAAGCACTCTAC. The mRNA made from this template will have complementary base pairing with uracil (U) instead of thymine (T). D. ACTATCTTCGTGAGATG By examining the DNA sequence, we can see that there is a -10 consensus sequence near the 3' end of the fragment. If we then count downstream, the +1 initiation site is the T immediately following the sequence AAT.

C

Cortisol passes through the plasma membrane to initiate signaling inside a cell. Which of the following correctly describes how the structures of cortisol and the plasma membrane allow this to occur? A. Cortisol is a protein. Proteins are derived from cholesterol, which makes them lipophilic (and hydrophobic). Therefore, cortisol can pass through the phospholipid membrane to initiate intracellular signaling. B. Cortisol is a steroid. Steroids are derived from carbohydrates, which makes them hydrophilic (and lipophobic). Therefore, cortisol can pass through the phospholipid membrane to initiate intracellular signaling. C. Cortisol is a steroid. Steroids are derived from cholesterol, which makes them lipophilic (and hydrophobic). Therefore, cortisol can pass through the phospholipid membrane to initiate intracellular signaling. D. Cortisol is a protein. Proteins are derived from cholesterol, which makes them hydrophilic (and lipophobic). Therefore, cortisol can pass through the phospholipid membrane to initiate intracellular signaling. E. Cortisol is a nucleic acid. Nucleic acids are derived from cholesterol, which makes them lipophilic (and hydrophobic). Therefore, cortisol can pass through the phospholipid membrane to initiate intracellular signaling.

C

During an experimental procedure, you apply a chemical to a cell. The cell responds to the chemical application approximately 30 minutes later. Part of the cellular response includes the proliferation of critical signaling proteins inside the cell. Which of the following best describes the signal transduction pathway that occurred? A. Activation of membrane bound receptor that initiates a signal transduction pathway that includes transcription factors. B. Activation of G protein linked receptor that initiates a second messenger system that can alter the cells metabolism. C. All of these describe a signal transduction pathway that occurred. D. Initiation of disassociation of peripheral proteins that go on to start the phosphorylation of additional proteins in a signal transduction pathway. E. Activation of a receptor from the extracellular space that initiates the disassociation of G protein subunits.

C

In a fly, the transcription rates of a gene that codes for an enzyme vary tremendously among different cell types. The most likely reason for these differences is that different cell types have different: A. promoters. B. inducers. C. transcription factors present. D. silencers. E. enhancers.

C

Suppose that in a certain population of lizards, the probability of the birth of a male is 0.6. Assume that the sex of each individual is independently determined. What is the probability that a clutch of three will be all male? A. 0.064 B. 0.125 C. 0.216 D. 0.25 E. 0.36

C

Which of the following DNA repair systems operates together with DNA replication? A. Excision B. Mismatch C. Proofreading D. All of the above E. None of the above

C

Which of the following correctly describes processes that lead to variation in offspring with the same parents? A. Crossover occurs in prophase I between non-sister homologous chromosomes. B. Random alignment during metaphase I leads to gametes that have a mixture of maternal and paternal chromosomes. C. all of these describe processes that lead to variation in offspring with the same parents. D. Segments of DNA are exchanged between maternally derived and paternally derived chromosomes, and new gene combinations are formed. E. Fertilization is random, in that any two gametes can fuse.

C

Which of the following correctly describes which categories of amino acid would you expect to find on a soluble protein's surface and those you would you expect to find in the interior? A. In membrane proteins, nonpolar and hydrophobic amino acid sidechains associate with the hydrophobic tails of phospholipids, while polar and charged amino acid side chains interact with the polar head groups or with the aqueous solution. B. Sometimes, positively and negatively charged amino acid side chains interact with one another in the interior of a protein, and polar or charged amino acid side chains that interact with a ligand can be found in the ligand binding pocket. C. All of these statements correctly describes which categories of amino acid would you expect to find on a soluble protein's surface and those you would you expect to find in the interior. D. Polar and charged amino acid residues are more likely to be found on the surface of soluble proteins where they can interact with water, and nonpolar amino acid side chains are more likely to be found in the interior where they are sequestered from water. E. None of these correctly describes which categories of amino acid would you expect to find on a soluble protein's surface and those you would you expect to find in the interior.

C

Which of the following degrade protein stability, which increases its reactivity? A. Phosphorylation decreases protein stability B. Adding a phosphate decreases protein stability C. Both a and b are correct D. NADH decreases protein stability E. NAD+ decreases protein stability

C

Which of the following describes why insulin signaling is slower than dopamine signaling? A. Because insulin acts like an endocrine signal that is transported to neighboring cells whereas dopamine acts like a paracrine signal that is transported to distant cells. B. Because insulin is a protein that can be carried through the bloodstream easily because it is hydrophilic whereas dopamine is hydrophobic and needs a carrier protein to be transported. C. Because insulin acts like an endocrine signal that is transported through the bloodstream and travels greater distances whereas dopamine acts like a paracrine signal that is transported to nearby neighboring cells. D. Because insulin acts like a paracrine signal that is transported through the bloodstream and travels greater distances whereas dopamine acts like an endocrine signal that is transported to nearby neighboring cells. E. Because dopamine is a protein that can be carried through the bloodstream easily because it is hydrophilic whereas insulin is hydrophobic and needs a carrier protein to be transported.

C

Which of the following is not one of the events required for cell division? A. Segregation B. Cytokinesis C. Crossing over D. DNA replication E. All of the above are required for cell division.

C

A nondisjunction that has occurred due to homologous chromosomes or sister chromatids failing to separate during meiosis I, resulting in a decrease in chromosome number.

C and D

Why can some insects walk on water?

Carbon is a unique versatile element that can form covalent bonds with four other carbons or with four different atoms to complete its octet. The covalent bonds can be polar or nonpolar. This property of carbon makes it a versatile and a unique element that can serve as the backbone or basic structural component of the various macromolecules that make up living organisms and are important for life.

What steps are necessary for Cdk to become fully active?

Cdk must bind to a cyclin, and it must be phosphorylated in the correct position to become fully active.

Injecting a potassium solution into a person's blood is lethal. Capital punishment and euthanasia utilize this method in their subjects. Why do you think a potassium solution injection is lethal?

Cells typically have a high concentration of potassium in the cytoplasm and are bathed in a high concentration of sodium. Injection of potassium dissipates this electrochemical gradient. In the heart muscle, the sodium/potassium potential is responsible for transmitting the signal that causes the muscle to contract. When this potential is dissipated, the signal can't be transmitted, and the heart stops beating. Potassium injections are also used to stop the heart from beating during surgery.

What are the similarities and differences between the structures of centrioles and flagella?

Centrioles and flagella are alike in that they are made up of microtubules. In centrioles, two rings of nine microtubule "triplets" are arranged at right angles to one another. This arrangement does not occur in flagella.

How does citrate from the citric acid cycle affect glycolysis?

Citrate can inhibit phosphofructokinase by feedback regulation.

In eukaryotic cells, DNA and RNA synthesis occur in a separate compartment from protein synthesis. In prokaryotic cells, both processes occur together. What advantages might there be to separating the processes? What advantages might there be to having them occur together?

Compartmentalization enables a eukaryotic cell to divide processes into discrete steps so it can build more complex protein and RNA products. But there is an advantage to having a single compartment as well: RNA and protein synthesis occurs much more quickly in a prokaryotic cell.

Use the probability method to calculate the genotypes and genotypic proportions of a cross between AABBCc and Aabbcc parents.

Considering each gene separately, the cross at A will produce offspring of which half are AA and half are Aa; B will produce all Bb; C will produce half Cc and half cc. Proportions then are (1/2) × (1) × (1/2), or 1/4 AABbCc; continuing for the other possibilities yields 1/4 AABbcc, 1/4 AaBbCc, and 1/4 AaBbcc. The proportions therefore are 1:1:1:1.

Part of cortisol's role in the body involves passing through the plasma membrane to initiate signaling inside a cell. Describe how the structures of cortisol and the plasma membrane allow this to occur.

Cortisol is a small, generally hydrophobic molecule, while the phospholipids that create plasma membranes have a hydrophilic head and hydrophobic tails. Since cortisol is hydrophobic, it can interact with the sequestered tails of the phospholipids in the center of the plasma membrane. This, along with its small size, allows cortisol to move through the plasma membrane to the inside of the cell.

Does crossing over occur during prophase II? From an evolutionary perspective, why is this advantageous?

Crossing over does not occur during prophase II; it only occurs during prophase I. In prophase II, there are still two copies of each gene, but they are on sister chromatids within a single chromosome (rather than homologous chromosomes as in prophase I). Therefore, any crossover event would still produce two identical chromatids. Because it is advantageous to avoid wasting energy on events that will not increase genetic diversity, crossing over does not occur.

An individual cell can require energy in much the same way as whole organisms require energy. Which of the following cellular functions requires energy in a similar manner to how a whole organism would require energy? A. Sodium potassium pump B. Secondary active transport C. Active transport D. All of these are a correct example E. The preparatory phase of glycolysis

D

During ATP synthesis, a redox agent helps the process of chemiosmosis, which involves which of the following? A. the movement of glucose through the cell membrane B. the movement of hydrogen atoms across a mitochondrial membrane C. the movement of electrons across the mitochondrial membrane D. the movement of hydrogen ions across a mitochondrial membrane E. the movement of electrons across the cell membrane

D

Imagine that the DNA sequence adjacent to position 90 on the diagram below functions as an origin of replication. What is the 11bp sequence and the orientation of the RNA primer that would bind to the top strand between base pair position 80 and 90 during DNA replication? A. 5' AAUAGUUCGA 3' B. 3' CGTACGCATGT 5' C. 5' CGTACGCATGT 3' D. 3' CGUACGCAUGU 5' E. 5' AATAGTTCGA 3' F. 3' AATAGTTCGA 5' G. 5' CGUACGCAUGU 3'

D

In which of the following disorders/diseases would be related to a problem with the Golgi apparatus? A. Protein assembly disorder B. Disorder affecting enzymatic breakdown of cellular waste products C. All of these would be related to a problem with the Golgi apparatus D. Congenital disorders of glycosylation (i.e. adding additional groups to protein) E. Genetic disorder affecting protein synthesis

D

Plant species that are predominantly comprised of chloroplasts with only chlorophyll a and chlorophyll b will undergo photosynthesis the best under which of the following conditions? A. light in the green part of the spectrum B. light in the orange part of the spectrum C. infrared light D. light in the blue part of the spectrum E. ultraviolet light

D

Tay-Sachs disease is a genetic disorder that results in the destruction of neurons due to a buildup of sphingolipids in the cells. Which organelle/structure is malfunctioning in Tay-Sachs? A. ribosomes B. nucleus C. Smooth Endoplasmic Reticulum D. lysosome E. plasma membrane

D

The adaptor between mRNA and a protein is: Answers: A. a promoter. B. RNA polymerase. C. DNA polymerase. D. tRNA. E. DNA

D

The enzyme alcohol dehydrogenase (ADH) carries out the conversion of alcohol to acetaldehyde. In the reaction below, ethanol is converted to acetaldehyde. Here, NAD+ / NADH serves as a(n)__________________________. A. all of these are correct B. enzyme C. inorganic cofactor D. coenzyme E. prosthetic group

D

The first five steps of glycolysis are best described by which of the following? A. An anabolic reaction in which a six carbon structure is divided. B. An endergonic reaction in which a six carbon structure is divided. C. A catabolic reaction in which glucose is converted to pyruvate D. An exergonic reaction in which a six carbon structure is divided. E. A metabolic reaction in which glucose is converted to pyruvate

D

The second meiotic division of meiosis: A. returns the chromosome number to diploid before fertilization. B. allows for crossing over and random distribution of chromosomes. C. reduces cell size by dividing the cytoplasm in half. D. prevents chromosome copies from doubling at each fertilization. E. is a step required for fertilization to take place.

D

The smallest unit of biological structure that meets the functional requirements of "living" is the: A. macromolecule B. organelle C. organ D. cell E. atom

D

What is the template DNA strand that would be used for transcription? A. 5ʹ-GTCTATGCATTA-3ʹ B. 5ʹ-CAGATACGTAAT-3ʹ C. 3ʹ-GTCTATGCATTA-5ʹ D. 3ʹ-CAGATACGTAAT-5ʹ E. 5ʹ-GUCUAUGCAUUA-3ʹ

D

When a large number of bacteria are present, which of the following can occur? A. Mating factor binds to cell surface receptors and signals the bacteria to form a biofilm B. Mating factors bind to cell surface receptors and bacteria form a colony C. Quorum sensing occurs triggered through mating factors D. Quorum sensing occurs triggered through autoinducers E. Autoinducers bind to cell surface receptors and initiate bacterial isolation.

D

Which of the following correctly describes ionic bonds differences from covalent bonds? A. There are no differences between ionic and covalent bond other than the fact that ionic bonds can be polar or non-polar. B. Covalent bonds are created between ions. The electrons are not shared between the atoms, but rather are associated more with one ion than the other. Covalent bonds are strong bonds, but are weaker than ionic bonds, meaning it takes less energy to break an covalent bond compared with a ionic one. C. Both bonds are created between ions. In both bonds, the electrons are not shared between the atoms, but rather are associated more with one ion than the other. D. Ionic bonds are created between ions. The electrons are not shared between the atoms, but rather are associated more with one ion than the other. Ionic bonds are strong bonds, but are weaker than covalent bonds, meaning it takes less energy to break an ionic bond compared with a covalent one. E. Covalent bonds are weaker than ionic bonds, but it does take less energy to break an ionic bond compared with a covalent one.

D

Which of the following correctly describes the general conditions that must be met at each of the main cell cycle checkpoints? A. At the G2 checkpoint, DNA is checked to ensure that all chromosomes were duplicated and that there are no mistakes in newly synthesized DNA. B. The G1 checkpoint monitors adequate cell growth, the state of the genomic DNA, adequate stores of energy, and materials for S phase. C. At the G2 checkpoint, cell size and energy reserves are evaluated. D. all of these correctly describes the conditions that must be met at the three cell cycle checkpoints. E. The M checkpoint confirms the correct attachment of the mitotic spindle fibers to the kinetochores.

D

Which of the following correctly describes the role electrons play in dehydration synthesis? A. In a dehydration synthesis reaction, the hydrogen of one monomer combines with the carbon of another monomer, which uses a molecule of water. This creates an opening in the outer shells of atoms in the monomers, which can share electrons and form covalent bonds. B. In a dehydration synthesis reaction, the hydrogen of one monomer combines with the carbon group of another monomer, releasing a molecule of water. This creates an opening in the outer shells of atoms in the monomers, which can share electrons and form ionic bonds. C. None of these correctly describe the role electrons play in dehydration synthesis D. In a dehydration synthesis reaction, the hydrogen of one monomer combines with the hydroxyl group of another monomer, releasing a molecule of water. This creates an opening in the outer shells of atoms in the monomers which share electrons and form covalent bonds E. In a dehydration synthesis reaction, the hydrogen of one monomer combines with the hydroxyl group of another monomer, which uses a molecule of water. This creates an opening in the outer shells of atoms in the monomers, which can share electrons and form ionic bonds.

D

Which of the following describe the difference between rho-dependent and rho-independent termination of transcription in prokaryotes? A. Rho-dependent termination is controlled by a stop sequence, which creates a hairpin loop. Near the end of the gene, the polymerase stalls at a run of G nucleotides on the DNA template. The rho protein collides with the polymerase and releases mRNA from the transcription bubble. Rho-independent termination is controlled by a protein. As the polymerase nears the end of the gene being transcribed, it encounters a stop protien that causes the polymerase to stall. B. Rho-dependent termination is controlled by the complimentary RNA strand. Near the end of the gene, the polymerase stalls at a run of G nucleotides on the RNA template. Rho-independent termination is controlled by specific sequences in the DNA template strand. As the polymerase nears the end of the gene being transcribed, it encounters a region rich in C-G nucleotides. This creates an mRNA hairpin that causes the polymerase to stall. C. Rho-dependent termination is controlled by the rho protein, which tracks in front of the polymerase. Near the end of the gene, the rho protein stops and the polymerase stalls. The polymerase collides behind the rho protein and releases mRNA from the transcription bubble. Rho-independent termination is controlled by specific sequences in the RNA complementary strand. As the polymerase nears the end of the RNA being transcribed, it encounters a region rich in C-G nucleotides. This creates an mRNA hairpin that causes the polymerase to stall. D. Rho-dependent termination is controlled by the rho protein, which tracks along behind the polymerase on the growing mRNA chain. Near the end of the gene, the polymerase stalls at a run of G nucleotides on the DNA template. The rho protein collides with the polymerase and releases mRNA from the transcription bubble. Rho-independent termination is controlled by specific sequences in the DNA template strand. As the polymerase nears the end of the gene being transcribed, it encounters a region rich in C-G nucleotides. This creates an mRNA hairpin that causes the polymerase to stall.

D

Which of the following describes one advantage and one disadvantage to the eukaryotic genome packaging compared to the prokaryotes? A. Advantage: The linear arrangement of the eukaryotic chromosome allows more DNA to be packed by tightly winding it around histones. Disadvantage: Maintaining more genetic material requires more energy. B. Advantage: More genetic material in the eukaryote means that the organism can encode more information into a single cell. Disadvantage: More genetic material introduces the possibility for more errors C. Advantage: The linear arrangement of the eukaryotic chromosome allows more DNA which will eventually allowed some eukaryotes to develop into multicellular organisms with cell specialization. Disadvantage: Maintaining more genetic material requires more complexity to the transcription process. D. A, B and C are all correct E. A and B are correct

D

Which of the following is correct with respect to electrochemical gradients? A. They are important for cells to function correctly B. Equilibrium of the ions in the gradient is determine by both concentration of the ion and electrical charge of the ion C. They can be a source of potential energy in the cell D. All of these are correct E. Only B and C are correct

D

Which of the following sequences is correct and represents the hierarchy of biological organization from the most inclusive to the least complex level? A. biosphere, ecosystem, community, population, organism B. biosphere, community, organism, cells, atoms C. community, population, organism, organs, tissues D. population, organism, organs, tissues, cells E. community, population, organism, organs, cells

D

Which of the following statements about meiosis and mitosis is true? A. The products of mitosis can be different from one another, while the products of meiosis are all the same (except for rare mutations). B. DNA replication occurs only in mitosis. C. DNA replication occurs only in meiosis. D. The products of meiosis can be different from one another, while the products of mitosis are all the same (except for rare mutations). E. Mitosis and meiosis are the same process.

D

Which of the following uses pyruvate as a starting substrate? A. Citric acid cycle B. Glycolysis C. Calvin Cycle D. Fermentation E. Electron transport chain

D

If mRNA is complementary to the DNA template strand and the DNA template strand is complementary to the DNA non template strand, then why are base sequences of mRNA and the DNA non template strand not identical? Could they ever be?

DNA is different from RNA in that T nucleotides in DNA are replaced with U nucleotides in RNA. Therefore, they could never be identical in base sequence.

Describe two ways that decreasing temperature would affect the rate of diffusion of molecules across a cell's plasma membrane.

Decreasing temperature will decrease the kinetic energy in the system. A lower temperature means less energy in the molecules, so they will move at a slower speed. Lowering temperature also decreases the kinetic energy of the molecules in the plasma membrane, compressing them together. This increases the density of the plasma membrane, which slows diffusion into the cell.

The same second messengers are used in many different cells, but the response to second messengers is different in each cell. How is this possible?

Different cells produce different proteins, including cell-surface receptors and signaling pathway components. Therefore, they respond to different ligands, and the second messengers activate different pathways. Signal integration can also change the end result of signaling.

Pseudomonas infections are very common in hospital settings. Why would it be important for doctors to determine the bacterial load before treating an infected patient?

Doctors would need to determine if the patient is simply infected with free bacteria, or has developed a biofilm. Biofilms of Pseudomonas aeruginosa have a different pattern of gene expression than free bacteria, leading to increased virulence and resistance to many antibiotics.

Briefly describe the events that occur in each phase of interphase.

During G1, the cell increases in size, the genomic DNA is assessed for damage, and the cell stockpiles energy reserves and the components to synthesize DNA. During the S phase, the chromosomes, the centrosomes, and the centrioles (animal cells) duplicate. During the G2 phase, the cell recovers from the S phase, continues to grow, duplicates some organelles, and dismantles other organelles.

Describe the process that results in the formation of a tetrad.

During the meiotic interphase, each chromosome is duplicated. The sister chromatids that are formed during synthesis are held together at the centromere region by cohesin proteins. All chromosomes are attached to the nuclear envelope by their tips. As the cell enters prophase I, the nuclear envelope begins to fragment and the proteins holding homologous chromosomes locate each other. The four sister chromatids align lengthwise, and a protein lattice called the synaptonemal complex is formed between them to bind them together. The synaptonemal complex facilitates crossover between nonsister chromatids, which is observed as chiasmata along the length of the chromosome. As prophase I progresses, the synaptonemal complex breaks down and the sister chromatids become free, except where they are attached by chiasmata. At this stage, the four chromatids are visible in each homologous pairing and are called a tetrad.

A dominant allele (K) is necessary for normal hearing. A dominant allele (M) on a different locus results in deafness no matter which other alleles are present. If a kkMm individual is crossed with a Kkmm individual, _______ percent of the offspring will be deaf. A. 0 B. 25 C. 50 D. 67 E. 75

E

A patient is experiencing increased concentration of carbon dioxide in the blood. Which of the following best describes what would happen to the activity of many (not all) enzymes in the respiratory system? A. The blood pH would increase, which would alter the activity of these enzymes, making them more effective. B. The blood temperature would increase, which would alter the activity of these enzymes, making them more effective. C. The blood pH would increase, which would alter the activity of these enzymes, making them less effective. D. The blood temperature would decrease, which would alter the activity of these enzymes, making them less effective. E. The blood pH would decrease, which would alter the activity of these enzymes, making them less effective.

E

Amino acids have a generic structure with different carbon-based side chains. Which of the following correctly describes how the structure of amino acids allows them to be linked into long peptide chains to form proteins? A. Since both ends of the molecule can participate in condensation reactions, peptide bonds can be made in both directions to create a long amino acid chain. B. Amino acids can be linked into long chains through dehydration synthesis. C. Amino acids can be linked into long chains through condensation reactions. D. One of the hydrogen atoms bonded to the nitrogen atom of an amino acid reacts with the -OH group attached to the terminal carbon on another amino acid. E. All of these statements correctly describes how the structure of amino acids allows them to be linked into long peptide chains to form proteins

E

An environmental change that leads to the production of less than the expected amount of cyclin would tend to _______ the activity of the Cdk. This would lead to _______ phosphorylation of the target protein, which in turn would lead to _______ cells being stopped at the checkpoint. A. increase; increased; more B. increase; decreased; fewer C. increase: decreased; fewer D. decrease; increased; fewer E. decrease; decreased; more

E

Because of the redundancy in the genetic code, many mutations in the coding regions of genes are: A. transitions. B. transversions. C. inversions. D. somatic mutations. E. silent mutations.

E

Chemotherapy drugs such as vincristine (derived from Madagascar periwinkle plants) and colchicine (derived from autumn crocus plants) disrupt mitosis by interfering with microtubule assembly and disassembly. Which of the following best describes how this would effect cell division? A. The mitotic spindle is formed of nucleosomes, which are polymers of the protein histones and nucleosides; therefore, it is the nucleosomes that is disrupted by these drugs. Without a functional nucleosomes, the chromosomes will not be sorted or separated during mitosis. The cell will arrest in mitosis and die. B. Disruption of microtubules will damage kinetochores in the centromere of the chromosomes. This will cause uneven division of sister chromatids in the newly dividing cell, which will cause the cell to stop division at the G1 checkpoints of interphase. C. By disrupting microtubules, the sister chromatids will fail to align along the metaphase plate, which will disrupt cytokinesis D. all of these are correct E. The mitotic spindle is formed of microtubules and therefore, it is the mitotic spindle that is disrupted by these drugs. Without a functional mitotic spindle, the chromosomes will not be sorted or separated during mitosis. The cell will arrest in mitosis and die.

E

Cyanide inhibits cytochrome c oxidase (i.e. cytochrome complex), a component of the electron transport chain. Which of the following would occur if cyanide toxicity occurs? A. The proton gradient would decrease B. The electron transport chain would no longer pump electrons into the intermembrane space. C. The pH of the intermembrane space would increase D. ATP synthesis would stop. E. All of these events would occur

E

How does the double helix structure of DNA support its role in encoding the genome? A. The sugar-phosphate backbone provides a template for DNA replication. B. Complementary base pairing allows for easy editing of both strands of DNA C. tRNA pairing with the template strand creates proteins encoded by the genome. D. All of these support the role of DNA in encoding the genome E. Complementary base pairing creates a very stable structure.

E

Imagine that a novel life-form is found deep within Earth's crust. Although its DNA is composed of the same four nucleotides as other life-forms, its codons are four bases in length. This organism could therefore be composed of _______ different amino acids. A. 4 B. 16 C. 64 D. 128 E. 256

E

In cocker spaniels, black color (B) is dominant over red (b), and solid color (S) is dominant over spotted (s). If the genes are unlinked and the offspring of BBss and bbss individuals are mated with each other, what fraction of their offspring will be black and spotted? A. 1/16 B. 9/16 C. 1/9 D. 3/16 E. 3/4

E

In transcription, which of the following occur? A. Synthesis of just protein B. Synthesis of RNA and protein C. Synthesis of just mRNA D. Synthesis of new DNA E. Synthesis of mRNA, tRNA and rRNA

E

In wild type Drosophila melanogaster, the order of genes on a chromosome is yellow, vermillion, forked, and Bobbed. Some individuals have a different gene order: yellow, Bobbed, forked, and vermillion. The individuals with the altered gene order most likely have a: A. missense mutation. B. nonsense mutation. C. reciprocal translocation. D. frame-shift mutation. E. chromosomal inversion.

E

Looking at this figure, how many electrons do elements in group 1 need to lose in order to achieve a stable electron configuration? How many electrons do elements in groups 14 and 17 need to gain to achieve a stable configuration? A. These elements do not need to gain or lose electrons B. Elements in group 1 need to lose no electron whereas elements in groups 14 need to gain four electrons and 17 need to lose one electron. C. Elements in group 1 need to lose one electron whereas elements in groups 14 do not need to gain or lose electrons but 17 need to gain one electron. D. Elements in group 1 need to lose one electron whereas elements in groups 14 need to lose four electrons and 17 need to lose one electron. E. Elements in group 1 need to lose one electron whereas elements in groups 14 need to gain four electrons and 17 need to gain one electron.

E

Microtubules that form the mitotic spindle tend to originate from or terminate in: A. centromeres and telomeres. B. euchromatin. C. centrioles and telomeres. D. the nuclear envelope. E. centrioles and kinetochores.

E

Mitochondrial syndrome is often inherited and is marked by mitochondria that are unable to completely convert glucose and oxygen into energy, which is essential for normal cell function. Symptoms include very poor growth, extreme developmental delays, and extreme muscle weakness. Which of the following explains why a person would be able to survive this syndrome? Answers: A. Because glycolysis does not occur in the mitochondria and small amount of energy is generated in this stage. B. Because some energy can be produced using lactic acid fermentation, which does not occur in the mitochondria. C. Both A and B are correct D. Because glycolysis and the citric acid cycle can occur outside the mitochondria and these processes produce adequate energy. E. B and D are both correct

E

One of the experimental treatments given to patients with Covid-19 is dexamethasone, which is a glucocorticoid (i.e. steroid) hormone. It decreases the bodies own response to the infection via a negative feedback system. Which of the following correctly contrasts how the receptor for dexamethasone is different from a receptor that binds a protein signal? A. Dexamethasone receptors are located in the cytoplasm whereas receptors for protein signals are on the plasma membrane. B. Dexamethasone binds to receptors to form a receptor complex that migrates to the nucleus and regulates DNA transcription whereas receptors for protein signals regulate second messenger systems and carrier or channel proteins. C. Dexamethasone receptors are located on the plasma membrane whereas receptors for protein signals are within the cell in the cytoplasm. D. Dexamethasone receptors will regulate second messenger systems and carrier or channel proteins whereas receptors for protein signals form a receptor complex that migrates to the nucleus and regulates DNA transcription. E. Both A and B are correct F. Both C and D are correct

E

Part of cortisol's role in the body involves passing through the plasma membrane to initiate signaling inside a cell. Which of the following correctly describes how the structure of cortisol and the plasma membrane allow this to occur? A. Cortisol is a small, generally hydrophobic molecule, while the phospholipids that create plasma membranes have a hydrophilic head and hydrophobic tails. Since cortisol is hydrophobic, it can interact with the sequestered tails of the phospholipids in the center of the plasma membrane. B. Cortisol is a small, generally lipophilic molecule, while the phospholipids that create plasma membranes have a hydrophilic head and hydrophobic tails. Since cortisol is lipophilic, it can interact with the sequestered tails of the phospholipids in the center of the plasma membrane. C. Cortisol is a small, generally hydrophilic molecule, while the phospholipids that create plasma membranes have a hydrophobic head and hydrophilic tails. Since cortisol is hydrophilic, it can interact with the sequestered tails of the phospholipids in the center of the plasma membrane. D. Cortisol is a small, generally hydrophobic molecule, while the phospholipids that create plasma membranes have a hydrophobic head and hydrophilic tails. Since cortisol is hydrophobic, it can interact with the sequestered tails of the phospholipids in the center of the plasma membrane. E. A and B are both correct

E

Phospholipids are important components of ________. A. the plasma membrane of eukaryotic cells B. the ring structure of steroids C. the waxy covering on leaves D. the plasma membrane of prokaryotic cells E. Both A and D are correct

E

Suppose members of a true-breeding strain of salamanders with yellow stripes are crossed with a true-breeding strain with red stripes. Red and yellow are two alleles of the same gene. The resulting offspring have both yellow stripes and red stripes. Which of the following dominance patterns is most likely? A. Red stripes are dominant to yellow stripes. B. Yellow stripes are dominant to red stripes. C. Red and yellow stripes exhibit incomplete dominance. D. Red and yellow stripes exhibit overdominance. E. Red and yellow stripes exhibit codominance.

E

Suppose that tall plants are crossed with short, and the progeny are medium height. The F1 plants are then crossed, and the progeny among the F2 have many more size classes (9) than what is typically seen in crosses involving a single gene (fewer than 4). Which of the following is the best explanation for this outcome? A. The existence of pleiotropic alleles B. Incomplete dominance C. Codominance D. Epistasis E. Multiple alleles

E

The monomers that make up proteins are called________. A. disaccharides B. glycosidic bonds C. both amino acids and nucleotides are correct D. nucleotides E. amino acids

E

The results of an experiment show that reaction A occurs at a rate in which 10 proteins are produced/ hour. You add a reagent that causes the reaction to produce 100 proteins/hour. Which of the following might you predict is contained within the reagent you added? A. A molecule with hydrogen (H+) that reacts within the components in the process B. Water molecules that are highly electronegative and hydrolyze ATP C. Water molecules that are highly electronegative and pull apart to donate H+ D. All of these would increase the rate of the reaction. E. An enzyme that catalyzed the reaction

E

Water moves from the kidney to the circulatory system using which of the following methods? A. Passive transport B. Secondary active transport C. Osmosis D. Facilitated passive transport E. A and C are both correct

E

When glucose levels in the blood are low, glucose transport into a cell occurs using which of the following? A. Passive transport B. Primary active transport C. Facilitated passive transport D. None of these E. Secondary active transport

E

Which correctly describes the major outcome of meiosis I vs. meiosis II? A. In meiosis I a cell reduces chromosome number from diploid to haploid whereas in meiosis II the chromosome number is conserved. B. In meiosis II a cell reduces chromosome number from diploid to haploid whereas in meiosis I the chromosome number is conserved. C. In meiosis I crossing over of homologous chromosomes and random assortment of chromosomes increase genetic diversity of haploid cells whereas in meiosis II the haploid cells distribute chromosome to the production of gametes. D. In meiosis II crossing over of homologous chromosomes and random assortment of chromosomes increase genetic diversity of haploid cells whereas in meiosis I the haploid cells distribute chromosome to the production of gametes. E. Both A and C are correct

E

Which of the following correctly compares saturated and unsaturated triglycerides? A. Unsaturated triglycerides contain no double bonds between carbon atoms; they are usually solid at room temperature. Saturated triglycerides contain at least one double bond between carbon atoms and are usually liquid at room temperature. B. Saturated triglycerides contain multiple double bonds between carbon atoms; they are usually liquid at room temperature and can form cis and trans isomers. Unsaturated triglycerides contain only one double bond between carbon atoms and are usually solid at room temperature. C. Saturated triglycerides contain no double bonds between carbon atoms; they are usually liquid at room temperature. Unsaturated triglycerides contain at least one double bond between carbon atoms and are usually solid at room temperature. D. Unsaturated triglycerides always contain multiple double bonds between carbon atoms; they are usually solid at room temperature and can form cis and trans isomers. Saturated triglycerides contain one double bond between carbon atoms and are usually liquid at room temperature. E. Saturated triglycerides contain no double bonds between carbon atoms; they are usually solid at room temperature. Unsaturated triglycerides contain at least one double bond between carbon atoms and are usually liquid at room temperature.

E

Which of the following correctly describes alterations in trophic interaction that could arise because of changes in plant physiology that occur with increased global temperatures? A. none of these are possible. Climate change can not affect trophic interactions in ecosystems. B. An increase in temperature may cause plants to open stomata, leading to an increase in plant populations. This will increase herbivore populations, which will decrease plant populations. The herbivore and plant populations will constantly cycle like this while omnivores and carnivores are unaffected. C. An increase in temperature may cause plants to open stomata, leading to an increase in plant populations. More plants would mean there is more food for herbivores. The herbivore populations will then also increase and therefore, carnivore populations will also increase. D. An increase in temperature may cause plants to close stomata, leading to a decrease in oxygen in the atmosphere. This will cause a decline in herbivore, omnivore and carnivore populations. E. Increased global temperatures can cause plants to close stomata, which will slow rate of photosynthesis and therefore glucose production. Fewer plants may survive, which will decrease herbivore populations and carnivore populations. Omnivores (including humans) will have to eat more plant-based diets as meat becomes scarce.

E

Which of the following is an example of a secondary protein structure? A. α-helix B. β-pleated sheet C. insulin D. A, B and C are correct E. A and B are correct

E

Which of the following is the correct order of events in mitosis? A. The kinetochore becomes attached to the mitotic spindle. Cohesin proteins break down and the sister chromatids separate. Sister chromatids line up at the metaphase plate. The nucleus reforms and the cell divides. B. Sister chromatids line up at the metaphase plate. The kinetochore becomes attached to the mitotic spindle. The nucleus reforms and the cell divides. Cohesin proteins break down and the sister chromatids separate. C. The centromere becomes attached to the centrosome. Chromosomes line up at the cell poles. Cohesin proteins break down and the chromosomes separate. The cell divides and the nucleus reforms. D. The kinetochore becomes attached to the cohesin proteins. Sister chromatids line up at the metaphase plate. The kinetochore breaks down and the sister chromatids separate. The nucleus reforms and the cell divides. E. The kinetochore becomes attached to the mitotic spindle. Sister chromatids line up at the metaphase plate. Cohesin proteins break down and the sister chromatids separate. The nucleus reforms and the cell divides.

E

Which of the following statements about epigenetics is false? A. One type of epigenetic change occurs via DNA methylation. B. One type of epigenetic change occurs via chromosomal protein alterations. C. Epigenetic changes can lead to gene expression changes. D. Epigenetic changes do not change the DNA sequence. E. All of the above are true; none is false.

E

Which of the following statements about sister chromatids is false? A. They arise by replication during S phase. B. They separate from each other during each mitotic anaphase. C. They usually contain identical versions of the same genetic information. D. They are joined during prophase I and metaphase I at their common centromere. E. They separate from each other during meiosis I.

E

Which type of bond represents the weakest chemical bond? A. all of these bonds are equal B. ionic bond C. van Der Waal interactions D. covalent bond E. hydrogen bond

E

Pathogenic E. coli have recently been shown to degrade tight junction proteins during infection. How would this provide an advantage to the bacteria?

E. coli infections generally cause food poisoning, meaning that the invading bacteria cross from the lumen of the gut into the rest of the body. Tight junctions hold the epithelial layer that lines the digestive tract together so that the material that crosses into the body is tightly regulated. One way E. coli can avoid this regulation is to destroy the tight junctions so that it can enter the body between the epithelial cells, rather than having to go through the cells.

If a cell developed a mutation in its MAP2K1 gene (encodes the MEK protein) that prevented MEK from being recognized by phosphatases, how would the EGFR signaling cascade and the cell's behavior change?

EGF binding to EGFR initiates a signaling cascade that activates protein kinases through phosphorylation. Active RAF phosphorylates MEK, activating MEK's kinase activity. If MEK cannot be dephosphorylated, the signaling cascade downstream of MEK will continue to be active after the EGF signal is gone. Therefore, the cell will continue to proliferate and be resistant to cell death (apoptosis).

In certain cancers, the GTPase activity of the RAS G-protein is inhibited. This means that the RAS protein can no longer hydrolyze GTP into GDP. What effect would this have on downstream cellular events?

ERK would become permanently activated, resulting in cell proliferation, migration, adhesion, and the growth of new blood vessels. Apoptosis would be inhibited.

Explain epistasis in terms of its Greek-language roots "standing upon."

Epistasis describes an antagonistic interaction between genes wherein one gene masks or interferes with the expression of another. The gene that is interfering is referred to as epistatic, as if it is "standing upon" the other (hypostatic) gene to block its expression.

Given the mRNA as in (a) from the question above, what are the first four amino acids of the resulting polypeptide (in the order they appear on the chain)? A. tryptophan,glutamine, lysine, leucine, B. tryosine, glutamine, asparagine, valine C. asparagine, isoleucine, trytophan,glutamine D. glutamine, tryptophan , leucine, valine E. histadine, valine, phenylalanine, methionine F. lysine, glutamine, leucine, tryptophan G. Serine, tyrosine, aspartic acid, cysteine

F

In order to be considered a paracrine signal, the chemical released from a cell must be prevented from straying too far. Which of the following mechanisms would accomplish this? A. Rapid uptake of the signal by neighboring cells B. Breakdown of the signal by extracellular enzymes C. Rapid binding by a carrier protein and moved into circulation D. Channel proteins that shuttle the signal into organelles E. All of these would accomplish this F. A and B would accomplish this.

F

Ionotropic receptors initiate second messenger systems

False

Receptors can be activated by only four signals: voltage, photons, temperature and chemicals

False

The making of a proton gradient is an exergonic reaction.

False

What accounts for the different number of ATP molecules that are formed through cellular respiration?

Few tissues except muscle produce the maximum possible amount of ATP from nutrients. The intermediates are used to produce needed amino acids, fatty acids, cholesterol, and sugars for nucleic acids. When NADH is transported from the cytoplasm to the mitochondria, an active transport mechanism is used, which decreases the amount of ATP that can be made. The electron transport chain differs in composition between species, so different organisms will make different amounts of ATP using their electron transport chains.

Imagine the Meselson and Stahl experiments had supported conservative replication instead of semi-conservative replication. What results would you predict to observe after two rounds of replication? Be specific regarding percent distributions of DNA incorporating 15N and 14N in the gradient.

Following two rounds of conservative replication, two bands would be detected after ultracentrifugation. A lower (heavier) band would be at the 15N density, and would comprise 25% of the total DNA. A second, higher (lighter) band would be at the 14N density, and would contain 75% of the total DNA.

Imagine if there were 200 commonly occurring amino acids instead of 20. Given what you know about the genetic code, what would be the shortest possible codon length? Explain.

For 200 commonly occurring amino acids, codons consisting of four types of nucleotides would have to be at least four nucleotides long, because 44 = 256. There would be much less degeneracy in this case.

Which of the following correctly describes glucose transport into a cell? A. Glucose can use passive transport via facilitated diffusion with a carrier protein when extracellular glucose levels are elevated. However, when extracellular glucose levels are low, it is transported using secondary active transport. B. Glucose can use passive transport via facilitated diffusion with a carrier protein when extracellular glucose levels are elevated. However, when extracellular glucose levels are low, it is transported using a symport system. C. Glucose can only be transported using passive transport via facilitated diffusion. D. Glucose can only be transported using secondary active transporters, which are a symport system. E. Glucose can only be transported using secondary active transporters, which are an antiport system. F. All of the above are correct G. Only A and B are correct

G

Which of the following requires an enzyme to complete the process? A. Digestion of fats B. Start of DNA transcription C. Converting 6 carbon sugar into two 3 carbon molecules D. Converting a molecule into its isomer E. Transfer of amino acids onto growing polypeptide chains F. Transferring electrons between molecules during redox reactions G. All of these require enzymes H. Only A, B and C require enzymes I. Only D, E, and F require enzymes

G

Describe the similarities and differences between glycogen and starch.

Glycogen and starch are polysaccharides. They are the storage form of glucose. Glycogen is stored in animals in the liver and in muscle cells, whereas starch is stored in the roots, seeds, and leaves of plants. Starch has two different forms, one unbranched (amylose) and one branched (amylopectin), whereas glycogen is a single type of a highly branched molecule.

How do organisms with haploid-dominant life cycles ensure continued genetic diversification in offspring without using a meiotic process to make gametes?

Haploid-dominant organisms undergo sexual reproduction by making a diploid zygote. The cells that make the gametes are derived from haploid cells, but the + and - mating types that produce the zygote are randomly combined. The zygote also undergoes meiosis to return to the haploid stage, so multiple steps add genetic diversity to haploid-dominant organisms.

Discuss why the following affect the rate of diffusion: molecular size, temperature, solution density, and the distance that must be traveled.

Heavy molecules move more slowly than lighter ones. It takes more energy in the medium to move them along. Increasing or decreasing temperature increases or decreases the energy in the medium, affecting molecular movement. The denser a solution is, the harder it is for molecules to move through it, causing diffusion to slow down due to friction. Living cells require a steady supply of nutrients and a steady rate of waste removal. If the distance these substances need to travel is too great, diffusion cannot move nutrients and waste materials efficiently to sustain life.

Compare and contrast a human somatic cell to a human gamete.

Human somatic cells have 46 chromosomes: 22 pairs and 2 sex chromosomes that may or may not form a pair. This is the 2n or diploid condition. Human gametes have 23 chromosomes, one each of 23 unique chromosomes, one of which is a sex chromosome. This is the n or haploid condition.

Why are hydrogen bonds and van der Waals interactions necessary for cells?

Hydrogen bonds and van der Waals interactions form weak associations between different molecules. They provide the structure and shape necessary for proteins and DNA within cells so that they function properly. Hydrogen bonds also give water its unique properties, which are necessary for life.

Which of the following describes the tonicity of the cell in C? A: 0.15 M NaCl (inside the cell), 0.15 M NaCl (outside the cell) B: 0.15 M NaCl (inside the cell), Pure H2O (outside the cell) C: 0.15 M NaCl (inside the cell), 1.5 M NaCl (outside the cell)

Hypertonic

Which of the following describes the tonicity of the cell in B? A: 0.15 M NaCl (inside the cell), 0.15 M NaCl (outside the cell) B: 0.15 M NaCl (inside the cell), Pure H2O (outside the cell) C: 0.15 M NaCl (inside the cell), 1.5 M NaCl (outside the cell)

Hypotonic

p53 can trigger apoptosis if certain cell-cycle events fail. How does this regulatory outcome benefit a multicellular organism?

If a cell has damaged DNA, the likelihood of producing faulty proteins is higher. The daughter cells of such a damaged parent cell would also produce faulty proteins that might eventually become cancerous. If p53 recognizes this damage and triggers the cell to self-destruct, the damaged DNA is degraded and recycled. No further harm comes to the organism. Another healthy cell is triggered to divide instead.

What is a possible result of a mutation in a kinase that controls a pathway that stimulates cell growth?

If a kinase is mutated so that it is always activated, it will continuously signal through the pathway and lead to uncontrolled growth and possibly cancer. If a kinase is mutated so that it cannot function, the cell will not respond to ligand binding.

What cell-cycle events will be affected in a cell that produces mutated (non-functional) cohesin protein?

If cohesin is not functional, chromosomes are not packaged after DNA replication in the S phase of interphase. It is likely that the proteins of the centromeric region, such as the kinetochore, would not form. Even if the mitotic spindle fibers could attach to the chromatids without packing, the chromosomes would not be sorted or separated during mitosis.

Nearly all organisms on Earth carry out some form of glycolysis. How does this fact support or not support the assertion that glycolysis is one of the oldest metabolic pathways?

If glycolysis evolved relatively late, it likely would not be as universal in organisms as it is. It probably evolved in very primitive organisms and persisted, with the addition of other pathways of carbohydrate metabolism that evolved later.

Outline the steps that lead to a cell becoming cancerous.

If one of the genes that produces regulator proteins becomes mutated, it produces a malformed, possibly non-functional, cell-cycle regulator, increasing the chance that more mutations will be left unrepaired in the cell. Each subsequent generation of cells sustains more damage. The cell cycle can speed up as a result of the loss of functional checkpoint proteins. The cells can lose the ability to self-destruct and eventually become "immortalized."

When Chargaff was performing his experiments, the tetranucleotide hypothesis, which stated that DNA was composed of GACT nucleotide repeats, was the most widely accepted view of DNA's composition. How did Chargaff disprove this hypothesis?

If the tetranucleotide hypothesis were true, then DNA would have to contain equal amounts of all four nucleotides (A=T=G=C). However, Chargaff demonstrated that A=T and G=C, but that the four nucleotides are not present in equal amounts.

What is the primary difference between a circular pathway and a linear pathway?

In a circular pathway, the final product of the reaction is also the initial reactant. The pathway is self-perpetuating, as long as any of the intermediates of the pathway are supplied. Circular pathways are able to accommodate multiple entry and exit points, thus being particularly well suited for amphibolic pathways. In a linear pathway, one trip through the pathway completes the pathway, and a second trip would be an independent event.

What role do electrons play in dehydration synthesis and hydrolysis?

In a dehydration synthesis reaction, the hydrogen of one monomer combines with the hydroxyl group of another monomer, releasing a molecule of water. This creates an opening in the outer shells of atoms in the monomers, which can share electrons and form covalent bonds.

People with trisomy 21 develop Down's syndrome. What law of Mendelian inheritance is violated in this disease? What is the most likely way this occurs?

In any trisomy disorder, a patient inherits 3 copies of a chromosome instead of the normal pair. This violates the Law of Segregation, and usually occurs when the chromosomes fail to separate during the first round of meiosis.

Compare the flow of energy with the flow of nutrients in a closed, sunny ecosystem consisting of a giraffe and a tree.

In the defined ecosystem, energy would radiate from the Sun, and be absorbed by the chlorophyll in the leaves of the tree. Photosynthesis would occur in the leaves, transforming the light energy into stored chemical energy in the covalent bonds of carbon molecules. The giraffe would eat the leaves of the tree, and digest the carbon molecules to release energy. In the same ecosystem, nutrients would cycle between the tree and the giraffe. The giraffe would consume oxygen and release carbon dioxide as its cells perform aerobic respiration to create chemical energy. The tree will consume the released carbon dioxide during photosynthesis to create its own stored chemical energy, and release oxygen as a by-product.

What are the genotypes of the individuals labeled 1, 2, and 3?

Individual 1 has the genotype aa. Individual 2 has the genotype Aa. Individual 3 has the genotype Aa.

Insulin is a hormone that regulates blood sugar by binding to its receptor, insulin receptor tyrosine kinase. How does insulin's behavior differ from steroid hormone signaling, and what can you infer about its structure?

Insulin's receptor is an enzyme-linked transmembrane receptor, as can be determined from the "tyrosine kinase" in its name. This receptor is embedded in the plasma membrane, and insulin binds to its extracellular (outer) surface to initiate intracellular signaling cascades. Normally, steroid hormones cross the plasma membrane to bind with intracellular receptors. These intracellular hormone-receptor complexes then interact directly with DNA to regulate transcription. This limits steroid hormones to be small, non-polar molecules so they can cross the plasma membrane. However, since insulin does not have to cross into the cell it could be large or polar (it is a small, polar molecule).

Glucose from digested food enters intestinal epithelial cells by active transport. Why would intestinal cells use active transport when most body cells use facilitated diffusion?

Intestinal epithelial cells use active transport to fulfill their specific role as the cells that transfer glucose from the digested food to the bloodstream. Intestinal cells are exposed to an environment with fluctuating glucose levels. Immediately after eating, glucose in the gut lumen will be high, and could accumulate in intestinal cells by diffusion. However, when the gut lumen is empty, glucose levels are higher in the intestinal cells. If glucose moved by facilitated diffusion, this would cause glucose to flow back out of the intestinal cells and into the gut. Active transport proteins ensure that glucose moves into the intestinal cells, and cannot move back into the gut. It also ensures that glucose transport continues to occur even if high levels of glucose are already present in the intestinal cells. This maximizes the amount of energy the body can harvest from food.

Why do ions have a difficult time getting through plasma membranes despite their small size?

Ions are charged, and consequently, they are hydrophilic and cannot associate with the lipid portion of the membrane. Ions must be transported by carrier proteins or ion channels.

How and why would the end products of photosynthesis be changed if a plant had a mutation that eliminated its photosystem II complex?

Knocking out photosystem II would eliminate the production of oxygen and ATP during photosynthesis. Photosystem II splits water into oxygen atoms, hydrogen protons that remain in the thylakoid lumen, and hydrogen-derived electrons that move from the reaction center into the electron transport chain. The transfer of an electron through the electron transport chain provides the energy to pump more protons into the thylakoid lumen to maintain a higher concentration of protons there. Moving protons across the thylakoid membrane back to the stroma provides the energy for ATP synthase to produce ATP. Without this proton gradient, ATP will not be synthesized.

Explain Griffith's transformation experiments. What did he conclude from them?

Live R cells acquired genetic information from the heat-killed S cells that "transformed" the R cells into S cells.

List some reasons why a cell that has just completed cytokinesis might enter the G0 phase instead of the G1 phase.

Many cells temporarily enter G0 until they reach maturity. Some cells are only triggered to enter G1 when the organism needs to increase that particular cell type. Some cells only reproduce following an injury to the tissue. Some cells never divide once they reach maturity.

A heterozygous pea plant produces violet flowers and yellow, round seeds. Describe the expected genotypes of the gametes produced by Mendelian inheritance. If all three genes are found on the same arm of one chromosome should a scientist predict that inheritance patterns will follow Mendelian genetics?

Mendelian inheritance would predict that all three genes are inherited independently. There are therefore 8 different gamete genotype possibilities: VYR, VYr, VyR, Vyr, vYR, vYr, vyR, vyr. If all three genes are found on the same chromosome arm, independent assortment is unlikely to occur because the genes are close together (linked).

A scientist sequencing mRNA identifies the following strand: CUAUGUGUCGUAACAGCCGAUGACCCG. What is the sequence of the amino acid chain this mRNA makes when it is translated?

Met Cys Arg Asn Ser Arg The first step to writing the amino acid sequence is to find the start codon AUG. Then, the nucleotide sequence is separated into triplets: CU AUG UGU CGU AAC AGC CGA UGA. We stop the translation at UGA because that triplet encodes a stop codon. When we convert these codons to amino acids, the sequence becomes Met Cys Arg Asn Ser Arg.

With regard to enzymes, why are vitamins necessary for good health? Give examples.

Most vitamins and minerals act as coenzymes and cofactors for enzyme action. Many enzymes require the binding of certain cofactors or coenzymes to be able to catalyze their reactions. Since enzymes catalyze many important reactions, it is critical to obtain sufficient vitamins and minerals from the diet and from supplements. Vitamin C (ascorbic acid) is a coenzyme necessary for the action of enzymes that build collagen, an important protein component of connective tissue throughout the body. Magnesium ion (Mg++) is an important cofactor that is necessary for the enzyme pyruvate dehydrogenase to catalyze part of the pathway that breaks down sugar to produce energy. Vitamins cannot be produced in the human body and therefore must be obtained in the diet.

Why is signaling in multicellular organisms more complicated than signaling in single-celled organisms?

Multicellular organisms must coordinate many different events in different cell types that may be very distant from each other. Single-celled organisms are only concerned with their immediate environment and the presence of other cells in the area.

What is the consequence of mutation of a mismatch repair enzyme? How will this affect the function of a gene?

Mutations are not repaired, as in the case of xeroderma pigmentosa. Gene function may be affected or it may not be expressed.

Errors in splicing are implicated in cancers and other human diseases. What kinds of mutations might lead to splicing errors? Think of different possible outcomes if splicing errors occur.

Mutations in the spliceosome recognition sequence at each end of the intron, or in the proteins and RNAs that make up the spliceosome, may impair splicing. Mutations may also add new spliceosome recognition sites. Splicing errors could lead to introns being retained in spliced RNA, exons being excised, or changes in the location of the splice site.

Given the mRNA as in (a), does translation end within the boxed TAATT sequence (labeled c below) at nucleotides 85-89?

No

Imagine that the DNA sequence adjacent to position 90 on the diagram below functions as an origin of replication. What is the 11bp sequence and the orientation of the RNA primer that would bind to the top strand between base pair position 80 and 90 during DNA replication? Would DNA synthesis from the primer in the above question be continuous if the origin were at base pair 90?

No

Imagine that this primer [in question 2 above] is being used as the replication fork moves to the left, will this primer be used to create the leading strand?

No

A doctor injects a patient with what the doctor thinks is an isotonic saline solution. The patient dies, and an autopsy reveals that many red blood cells have been destroyed. Do you think the solution the doctor injected was really isotonic?

No, it must have been hypotonic, as a hypotonic solution would cause water to enter the cells, thereby making them burst.

A scientist splices a eukaryotic promoter in front of a bacterial gene and inserts the gene in a bacterial chromosome. Would you expect the bacteria to transcribe the gene?

No. Prokaryotes use different promoters than eukaryotes.

If no activation energy were required to break down sucrose (table sugar), would you be able to store it in a sugar bowl?

No. We can store chemical energy because of the need to overcome the barrier to its breakdown.

Which part of the light-independent reactions would be affected if a cell could not produce the enzyme RuBisCO?

None of the cycle could take place, because RuBisCO is essential in fixing carbon dioxide. Specifically, RuBisCO catalyzes the reaction between carbon dioxide and RuBP at the start of the cycle.

Chronic lymphocytic leukemia patients often harbor nonsense mutations in their spliceosome machinery. Describe how this mutation of the spliceosome would change the final location and sequence of a pre-mRNA.

Nonsense spliceosome mutations would eliminate the splicing step of mRNA processing, so the mature mRNAs would retain their introns and be perfectly complementary to the entire DNA template sequence. However, the mRNAs would still undergo addition of the 5' cap and poly-A tail, and therefore each has the potential to be exported to the cytoplasm for translation.

A normal mRNA that reads 5' -UGCCAUGGUAAUAACACAUGAGGCCUGAAC- 3' has an insertion mutation that changes the sequence to 5'-UGCCAUGGUUAAUAACACAUGAGGCCUGAAC- 3'.Translate the original mRNA and the mutated mRNA, and explain how insertion mutations can have dramatic effects on proteins. (Hint: Be sure to find the initiation site.)

Original mRNA: 5' -UGCC AUG GUA AUA ACA CAU GAG GCC UGA AC- 3'; Translation: Met - Val - Ile - Thr - His - Glu - Ala; Mutated mRNA: 5' -UGCC AUG GUU AAU AAC ACA UGA GGCCUGAAC- 3'; Translation: Met - Val - Asn - Asn - Thr; Insertion mutations can have dramatic effects on proteins because they shift the reading frame for the codons. This changes the amino acids encoded by the mRNA, and can introduce premature start or stop sites.

How can a cell use an extracellular peripheral protein as the receptor to transmit a signal into the cell?

Peripheral proteins can bind to other molecules in the extracellular space. However, they cannot directly transmit a signal to the inside of the cell since they do not have a transmembrane domain (region that goes through the plasma membrane to the inside of the cell). They must associate with integral membrane proteins in order to pass the signal to the inside of the cell.

Does physical exercise involve anabolic and/or catabolic processes? Give evidence for your answer.

Physical exercise involves both anabolic and catabolic processes. Body cells break down sugars to provide ATP to do the work necessary for exercise, such as muscle contractions. This is catabolism. Muscle cells also must repair muscle tissue damaged by exercise by building new muscle. This is anabolism.

Which categories of amino acid would you expect to find on the surface of a soluble protein, and which would you expect to find in the interior? What distribution of amino acids would you expect to find in protein embedded in a lipid bilayer?

Polar and charged amino acid residues (the remainder after peptide bond formation) are more likely to be found on the surface of soluble proteins where they can interact with water, and nonpolar (e.g., amino acid side chains) are more likely to be found in the interior where they are sequestered from water. In membrane proteins, nonpolar and hydrophobic amino acid side chains associate with the hydrophobic tails of phospholipids, while polar and charged amino acid side chains interact with the polar head groups or with the aqueous solution. However, there are exceptions. Sometimes, positively and negatively charged amino acid side chains interact with one another in the interior of a protein, and polar or charged amino acid side chains that interact with a ligand can be found in the ligand binding pocket.

Compare and contrast the roles of the positive cell-cycle regulators negative regulators.

Positive cell regulators such as cyclin and Cdk perform tasks that advance the cell cycle to the next stage. Negative regulators such as Rb, p53, and p21 block the progression of the cell cycle until certain events have occurred.

A scientist notices that a cancer cell line shows high levels of phosphorylated ERK in the absence of EGF. What are two possible explanations for the increase in phosphorylated ERK? Be specific in which proteins are involved.

Possible explanations: 1) EGFR dimer cannot separate. 2) An upstream mutation (in RAS, RAF, MEK) constitutively activates the signaling cascade. 3) ERK has a mutation that prevents it from binding to its phosphatase. 4) The cell has a mutation preventing the expression or function of the ERK-specific phosphatase.

Explain how the random alignment of homologous chromosomes during metaphase I contributes to the variation in gametes produced by meiosis.

Random alignment leads to new combinations of traits. The chromosomes that were originally inherited by the gamete-producing individual came equally from the egg and the sperm. In metaphase I, the duplicated copies of these maternal and paternal homologous chromosomes line up across the center of the cell. The orientation of each tetrad is random. There is an equal chance that the maternally derived chromosomes will be facing either pole. The same is true of the paternally derived chromosomes. The alignment should occur differently in almost every meiosis. As the homologous chromosomes are pulled apart in anaphase I, any combination of maternal and paternal chromosomes will move toward each pole. The gametes formed from these two groups of chromosomes will have a mixture of traits from the individual's parents. Each gamete is unique.

Rb and other proteins that negatively regulate the cell cycle are sometimes called tumor suppressors. Why do you think the name tumor suppressor might be appropriate for these proteins?

Rb and other negative regulatory proteins control cell division and therefore prevent the formation of tumors. Mutations that prevent these proteins from carrying out their function can result in cancer.

Rb is a negative regulator that blocks the cell cycle at the G1 checkpoint until the cell achieves a requisite size. What molecular mechanism does Rb employ to halt the cell cycle?

Rb is active when it is dephosphorylated. In this state, Rb binds to E2F, which is a transcription factor required for the transcription and eventual translation of molecules required for the G1/S transition. E2F cannot transcribe certain genes when it is bound to Rb. As the cell increases in size, Rb becomes phosphorylated, inactivated, and releases E2F. E2F can then promote the transcription of the genes it controls, and the transition proteins will be produced.

How does the extracellular matrix control the growth of cells?

Receptors on the cell surface must be in contact with the extracellular matrix in order to receive positive signals that allow the cell to live. If the receptors are not activated by binding, the cell will undergo apoptosis. This ensures that cells are in the correct place in the body and helps to prevent invasive cell growth as occurs in metastasis in cancer.

List the regulatory mechanisms that might be lost in a cell producing faulty p53.

Regulatory mechanisms that might be lost include monitoring of the quality of the genomic DNA, recruiting of repair enzymes, and the triggering of apoptosis.

In your own words, describe the difference between rho-dependent and rho-independent termination of transcription in prokaryotes.

Rho-dependent termination is controlled by the rho protein, which tracks along behind the polymerase on the growing mRNA chain. Near the end of the gene, the polymerase stalls at a run of G nucleotides on the DNA template. The rho protein collides with the polymerase and releases mRNA from the transcription bubble. Rho-independent termination is controlled by specific sequences in the DNA template strand. As the polymerase nears the end of the gene being transcribed, it encounters a region rich in C-G nucleotides. This creates an mRNA hairpin that causes the polymerase to stall right as it begins to transcribe a region rich in A-T nucleotides. Because A-U bonds are less thermostable, the core enzyme falls away.

If the nucleolus were not able to carry out its function, what other cellular organelles would be affected?

Ribosomes and Rough ER, because no ribosomes would be produced by the nucleolus. Therefore, protein synthesis would not take place, thus disrupting the whole functioning of cells.

You already know that ribosomes are abundant in red blood cells. In what other cells of the body would you find them in great abundance? Why?

Ribosomes are abundant in muscle cells as well because muscle cells are constructed of the proteins made by the ribosomes.

What advantage might biofilm production confer on the S. aureus inside the catheter?

S. aureus produces a biofilm because the higher cell density in the biofilm permits the formation of a dense surface that helps protect the bacteria from antibiotics.

Explain why sexual reproduction is beneficial to a population but can be detrimental to an individual offspring.

Sexual reproduction increases the genetic variation within the population, because new individuals are made by randomly combining genetic material from two parents. Because only fit individuals reach sexual maturity and reproduce, the overall population tends toward increasing fitness in its environment. However, there is always a possibility that the random combination creating the offspring's genome will actually produce an organism less fit for the environment than its parents were.

What are Okazaki fragments and how they are formed?

Short DNA fragments are formed on the lagging strand synthesized in a direction away from the replication fork. These are synthesized by DNA pol.

In your opinion, is the nuclear membrane part of the endomembrane system? Why or why not? Defend your answer.

Since the external surface of the nuclear membrane is continuous with the rough endoplasmic reticulum, which is part of the endomembrane system, then it is correct to say that it is part of the system.

Explain why not all microbes are harmful.

Some microbes are beneficial. For instance, E. coli bacteria populate the human gut and help break down fiber in the diet. Some foods such as yogurt are formed by bacteria.

Describe the general conditions that must be met at each of the three main cell-cycle checkpoints.

The G1 checkpoint monitors adequate cell growth, the state of the genomic DNA, adequate stores of energy, and materials for S phase. At the G2 checkpoint, DNA is checked to ensure that all chromosomes were duplicated and that there are no mistakes in newly synthesized DNA. Additionally, cell size and energy reserves are evaluated. The M checkpoint confirms the correct attachment of the mitotic spindle fibers to the kinetochores.

The sodium/calcium exchanger (NCX) transports sodium into and calcium out of cardiac muscle cells. Describe why this transporter is classified as secondary active transport.

The NCX moves sodium down its electrochemical gradient into the cell. Since sodium's electrochemical gradient is created by the Na+/K+ pump, a transport pump that requires ATP hydrolysis to establish the gradient, the NCX is a secondary active transport process.

Use a Punnett square to predict the offspring in a cross between a dwarf pea plant (homozygous recessive) and a tall pea plant (heterozygous). What is the phenotypic ratio of the offspring?

The Punnett square would be 2 × 2 and will have T and T along the top, and T and t along the left side. Clockwise from the top left, the genotypes listed within the boxes will be Tt, Tt, tt, and tt. The phenotypic ratio will be 1 tall: 1 dwarf.

Do you think that the EA for ATP hydrolysis is relatively low or high? Explain your reasoning.

The activation energy for hydrolysis is very low. Not only is ATP hydrolysis an exergonic process with a large −∆G, but ATP is also a very unstable molecule that rapidly breaks down into ADP + Pi if not utilized quickly. This suggests a very low EA since it hydrolyzes so quickly.

Prokaryotic cells are much smaller than eukaryotic cells. What advantages might small cell size confer on a cell? What advantages might large cell size have?

The advantage of a small cell size is that substances like organic molecules and ions can easily diffuse into the cells whereas wastes can easily diffuse out of the cells. As prokaryotic cells are significantly smaller than eukaryotic cells and do not have organelles, they do not need to expend as much energy getting substances across organelle membranes. The advantage of a large cell size is that larger cells are generally eukaryotic which means that they contain membrane-bound organelles, which can separate cellular processes, thereby enabling them to build more complex molecules.

What are the advantages and disadvantages of each of these types of microscopes?

The advantages of light microscopes are that they are easily obtained, and the light beam does not kill the cells. However, typical light microscopes are somewhat limited in the amount of detail they can reveal. Electron microscopes are ideal because you can view intricate details, but they are bulky and costly, and preparation for the microscopic examination kills the specimen.

What would happen if the intracellular domain of a cell-surface receptor was switched with the domain from another receptor?

The binding of the ligand to the extracellular domain would activate the pathway normally activated by the receptor donating the intracellular domain.

Explain how the formation of an adult human follows the cell theory.

The cell theory states: 1) All living things are made of cells. 2) Cells are the most basic unit of life. 3) New cells arise from existing cells. All humans are multicellular organisms whose smallest building blocks are composed of cells. Adult humans begin with the fusion of a male gamete cell with a female gamete cell to form a fertilized egg (single cell). That cell then divides into two cells, which then further divides into two more cells and so on, until all the cells of a human embryo are eventually made. As the embryo passes through all the developmental stages to make an adult human, the cells that are added arise from division of existing cells.

Antibiotics are medicines that are used to fight bacterial infections. These medicines kill prokaryotic cells without harming human cells. What part or parts of the bacterial cell do you think antibiotics target? Why?

The cell wall would be targeted by antibiotics as well as the bacteria's ability to replicate. This would inhibit the bacteria's ability to reproduce, and it would compromise its defense mechanisms.

In Section 12.3, "Laws of Inheritance," an example of epistasis was given for the summer squash. Cross white WwYy heterozygotes to prove the phenotypic ratio of 12 white: 3 yellow: 1 green that was given in the text.

The cross can be represented as a 4 × 4 Punnett square, with the following gametes for each parent: WY, Wy, wY, and wy. For all 12 of the offspring that express a dominant W gene, the offspring will be white. The three offspring that are homozygous recessive for w but express a dominant Y gene will be yellow. The remaining wwyy offspring will be green.

Explain how the extracellular matrix functions.

The extracellular matrix functions in support and attachment for animal tissues. It also functions in the healing and growth of the tissue.

What are the four types of RNA and how do they function?

The four types of RNA are messenger RNA, ribosomal RNA, transfer RNA, and microRNA. Messenger RNA carries the information from the DNA that controls all cellular activities. The mRNA binds to the ribosomes that are constructed of proteins and rRNA, and tRNA transfers the correct amino acid to the site of protein synthesis. microRNA regulates the availability of mRNA for translation.

Describe one of the reasons why the garden pea was an excellent choice of model system for studying inheritance.

The garden pea is sessile and has flowers that close tightly during self-pollination. These features help to prevent accidental or unintentional fertilizations that could have diminished the accuracy of Mendel's data.

What is the relationship between a genome, chromosomes, and genes?

The genome consists of the sum total of an organism's chromosomes. Each chromosome contains hundreds and sometimes thousands of genes, segments of DNA that code for a polypeptide or RNA, and a large amount of DNA with no known function.

Describe how the grey wolf population would be impacted by a volcanic eruption that spewed a dense ash cloud that blocked sunlight in a section of Yellowstone National Park.

The grey wolves are apex predators in their food web, meaning they consume smaller prey animals and are not the prey of any other animal. Blocking sunlight would prevent the plants at the bottom of the food web from performing photosynthesis. This would kill many of the plants, reducing the food sources available to smaller animals in Yellowstone. A smaller prey animal population means that fewer wolves can survive in the area, and the population of grey wolves will decrease.

How is the structure of the monosaccharide changed from one form to the other in the human body?

The human body switches carbohydrates between their aldose and ketose forms using a family of enzymes called isomerases. The ketose triose is called dihydroxyacetone, and has the oxygen double-bonded to the center carbon. The aldose is called glyceraldehyde, and can have the oxygen double-bonded to the first or third carbon of the molecule.

Why do phospholipids tend to spontaneously orient themselves into something resembling a membrane?

The hydrophobic, nonpolar regions must align with each other in order for the structure to have minimal potential energy and, consequently, higher stability. The fatty acid tails of the phospholipids cannot mix with water, but the phosphate "head" of the molecule can. Thus, the head orients to water, and the tail to other lipids.

Tremetol, a metabolic poison found in the white snake root plant, prevents the metabolism of lactate. When cows eat this plant, tremetol is concentrated in the milk they produce. Humans who consume the milk can become seriously ill. Symptoms of this disease, which include vomiting, abdominal pain, and tremors, become worse after exercise. Why do you think this is the case?

The illness is caused by lactate accumulation. Lactate levels rise after exercise, making the symptoms worse. Milk sickness is rare today but was common in the midwestern United States in the early 1800s.

Chemotherapy drugs such as vincristine (derived from Madagascar periwinkle plants) and colchicine (derived from autumn crocus plants) disrupt mitosis by binding to tubulin (the subunit of microtubules) and interfering with microtubule assembly and disassembly. Exactly what mitotic structure is targeted by these drugs and what effect would that have on cell division?

The mitotic spindle is formed of microtubules. Microtubules are polymers of the protein tubulin; therefore, it is the mitotic spindle that is disrupted by these drugs. Without a functional mitotic spindle, the chromosomes will not be sorted or separated during mitosis. The cell will arrest in mitosis and die.

What is the overall outcome of the light reactions in photosynthesis?

The outcome of light reactions in photosynthesis is the conversion of solar energy into chemical energy that the chloroplasts can use to do work (mostly anabolic production of carbohydrates from carbon dioxide).

Why are plasma membranes arranged as a bilayer rather than a monolayer?

The phospholipids organize themselves in a bilayer to hide their hydrophobic tail regions and expose the hydrophilic regions to water. This organization is spontaneous, meaning it is a natural process and does not require energy. This structure forms the layer that is the wall between the inside and outside of the cell. The most important property of the lipid bilayer is that it is a highly impermeable structure. Impermeable simply means that it does not allow molecules to freely pass across it. Only water and gases can easily pass through the bilayer. This property means that large molecules and small polar molecules cannot cross the bilayer, and thus the cell membrane, without the assistance of other structures.

In pea plants, purple flowers (P) are dominant to white flowers (p) and yellow peas (Y) are dominant to green peas (y). What are the possible genotypes and phenotypes for a cross between PpYY and ppYy pea plants? How many squares do you need to do a Punnett square analysis of this cross?

The possible genotypes are PpYY, PpYy, ppYY, and ppYy. The former two genotypes would result in plants with purple flowers and yellow peas, while the latter two genotypes would result in plants with white flowers with yellow peas, for a 1:1 ratio of each phenotype. You only need a 2 × 2 Punnett square (four squares total) to do this analysis because two of the alleles are homozygous.

Aquaporins are proteins embedded in the plasma membrane that allow water molecules to move between the extracellular matrix and the intracellular space. Based on its function and location, describe the key features of the protein's shape and the chemical characteristics of its amino acids.

The protein must form a channel in the plasma membrane that allows water into the cell since water cannot cross the plasma membrane by itself. Since aquaporins are embedded in the plasma membrane and connect with both the intracellular and extracellular spaces, it must be amphipathic like the plasma membrane. The top and bottom of the protein must contain charged or polar amino acids (hydrophilic) to interact with the aqueous environments. The exterior transmembrane region must contain non-polar amino acids (hydrophobic) that can interact with the phospholipid tails. However, the inside of this channel must contain hydrophilic amino acids since they will interact with the traveling water molecules.

Describe the differences in the four protein structures.

The sequence and number of amino acids in a polypeptide chain is its primary structure. The local folding of the polypeptide in some regions is the secondary structure of the protein. The three-dimensional structure of a polypeptide is known as its tertiary structure, created in part by chemical interactions such as hydrogen bonds between polar side chains, van der Waals interactions, disulfide linkages, and hydrophobic interactions. Some proteins are formed from multiple polypeptides, also known as subunits, and the interaction of these subunits forms the quaternary structure.

How does the sodium-potassium pump contribute to the net negative charge of the interior of the cell?

The sodium-potassium pump forces out three (positive) Na+ ions for every two (positive) K+ ions it pumps in, thus the cell loses a positive charge at every cycle of the pump.

How does the closing of the stomata limit photosynthesis?

The stomata regulate the exchange of gases and water vapor between a leaf and its surrounding environment. When the stomata are closed, the water molecules cannot escape the leaf, but the leaf also cannot acquire new carbon dioxide molecules from the environment. This limits the light-independent reactions to only continuing until the carbon dioxide stores in the leaf are depleted.

Describe the position of the transition state on a vertical energy scale, from low to high, relative to the position of the reactants and products, for both endergonic and exergonic reactions.

The transition state is always higher in energy than the reactants and the products of a reaction (therefore, above), regardless of whether the reaction is endergonic or exergonic.

Why is it impossible for humans to digest food that contains cellulose?

The β-1,4 glycosidic linkage in cellulose cannot be broken down by human digestive enzymes. Herbivores such as cows, koalas, and buffalos are able to digest grass that is rich in cellulose and use it as a food source because bacteria and protists in their digestive systems, especially in the rumen, secrete the enzyme cellulase. Cellulases can break down cellulose into glucose monomers that can be used as an energy source by the animal.

Describe the similarities and differences between the cytokinesis mechanisms found in animal cells versus those in plant cells.

There are very few similarities between animal cell and plant cell cytokinesis. In animal cells, a ring of actin fibers is formed around the periphery of the cell at the former metaphase plate (cleavage furrow). The actin ring contracts inward, pulling the plasma membrane toward the center of the cell until the cell is pinched in two. In plant cells, a new cell wall must be formed between the daughter cells. Due to the rigid cell walls of the parent cell, contraction of the middle of the cell is not possible. Instead, a phragmoplast first forms. Subsequently, a cell plate is formed in the center of the cell at the former metaphase plate. The cell plate is formed from Golgi vesicles that contain enzymes, proteins, and glucose. The vesicles fuse and the enzymes build a new cell wall from the proteins and glucose. The cell plate grows toward and eventually fuses with the cell wall of the parent cell.

Why have trans fats been banned from some restaurants? How are they created?

Trans fats are created artificially when hydrogen gas is bubbled through oils to solidify them. The double bonds of the cis conformation in the hydrocarbon chain may be converted to double bonds in the trans configuration. Some restaurants are banning trans fats because they cause higher levels of LDL, or "bad" cholesterol.

A sodium potassium pump is an example of symport

True

The diagram below shows a replication bubble within a DNA strand. A primer for DNA replication is 5'-GUACGUUG-3". There are multiple start/stop sites marked by letters and colors. Each letter corresponds to a color and is marked with a start-stop site and an arrow showing a direction in which replication would proceed. Which of the following correctly represents where the primer would anneal in the replication bubble and the direction of replication it would follow? Which is the correct anticodon on tRNA's that would recognize this mRNA? 5' AGATGCCGCACTGAC 3' 3' TCTACGGCGTGACTG 5'

UCUACGGCGUGACUG

Compare and contrast saturated and unsaturated triglycerides.

Unsaturated triglycerides contain one or more double bonds between carbon atoms and can be liquid or solid at room temperature depending upon the cis- or trans-configuration of the double bond, respectively. Saturated triglycerides do not contain double bonds between carbon atoms and are solids at room temperature. They have a high melting point whereas unsaturated TAGs have a low melting point.

Why is it more efficient to perform a test cross with a homozygous recessive donor than a homozygous dominant donor? How could the same information still be found with a homozygous dominant donor?

Using a homozygous recessive donor is more efficient because the genotype of the unknown parent can be determined in a single generation. If a homozygous dominant donor was used, the unknown genotype could still be determined. Instead of knowing the unknown genotype through the F1 phenotype, the F1 offspring would have to be self-crossed (as Mendel allowed his pea plants to self-pollinate) and the F2 generation phenotypes would be used to determine the unknown F0 genotype.

Why does water move through a membrane?

Water moves through a membrane in osmosis because there is a concentration gradient across the membrane of solute and solvent. The solute cannot effectively move to balance the concentration on both sides of the membrane, so water moves to achieve this balance.

Energy transfers take place constantly in everyday activities. Think of two scenarios: cooking on a stove and driving. Explain how the second law of thermodynamics applies to these two scenarios.

While cooking, food is heating up on the stove, but not all of the heat goes to cooking the food, some of it is lost as heat energy to the surrounding air, increasing entropy. While driving, cars burn gasoline to run the engine and move the car. This reaction is not completely efficient, as some energy during this process is lost as heat energy, which is why the hood and the components underneath it heat up while the engine is turned on. The tires also heat up because of friction with the pavement, which is additional energy loss. This energy transfer, like all others, also increases entropy.

A cell develops a mutation in its potassium channels that prevents the ions from leaving the cell. If the cell's aquaporins are still active, what will happen to the cell? Be sure to describe the tonicity and osmolarity of the cell.

Without functional potassium channels, the potassium ions that are pumped into the cell will accumulate. This increases the osmolarity inside the cell, creating a hypotonic solution. Since the plasma membrane is still selectively permeable to water by the aquaporins, water will flow into the cell. If the potassium concentration is high enough, enough water will eventually flow into the cell to lyse it.

In pea plants, round peas (R) are dominant to wrinkled peas (r). You do a test cross between a pea plant with wrinkled peas (genotype rr) and a plant of unknown genotype that has round peas. You end up with three plants, all which have round peas. From this data, can you tell if the round pea parent plant is homozygous dominant or heterozygous? If the round pea parent plant is heterozygous, what is the probability that a random sample of 3 progeny peas will all be round?

You cannot be sure if the plant is homozygous or heterozygous as the data set is too small: by random chance, all three plants might have acquired only the dominant gene even if the recessive one is present. If the round pea parent is heterozygous, there is a one-eighth probability that a random sample of three progeny peas will all be round.

A scientist introduces a mutation that makes the 60S ribosomal subunit nonfunctional in a human cell line. What would be the predicted effect on translation? a. Translation stalls after the initiation AUG codon is identified b. The ribosome cannot catalyze the formation of peptide bonds between the tRNAs in the A and P sites c. The ribosome cannot interact with mRNAs d. tRNAs cannot exit the E site of the ribosome

a

Assuming no gene linkage, in a dihybrid cross of AABB x aabb with AaBb F1 heterozygotes, what is the ratio of the F1 gametes (AB, aB, Ab, ab) that will give rise to the F2 offspring? a. 1:1:1:1 b. 1:3:3:1 c. 1:2:2:1 d. 4:3:2:1

a

At metaphase I, homologous chromosomes are connected only at what structures? a. chiasmata b. recombination nodules c. microtubules d. kinetochores

a

At which of the cell-cycle checkpoints do external forces have the greatest influence? a. G1 checkpoint b. G2 checkpoint c. M checkpoint d. G0 checkpoint

a

Bacteria that lack fimbriae are less likely to ________. a. adhere to cell surfaces b. swim through bodily fluids c. synthesize proteins d. retain the ability to divide

a

DNA replication involves unwinding two strands of parent DNA, copying each strand to synthesize complementary strands, and releasing the parent and daughter DNA. Which of the following accurately describes this process? a. This is an anabolic process b. This is a catabolic process c. This is both anabolic and catabolic d. This is a metabolic process but is neither anabolic nor catabolic

a

During the breakdown of polymers, which of the following reactions takes place? a. hydrolysis b. dehydration c. condensation d. covalent bond

a

Endocrine signals are transmitted more slowly than paracrine signals because ___________. a. the ligands are transported through the bloodstream and travel greater distances b. the target and signaling cells are close together c. the ligands are degraded rapidly d. the ligands don't bind to carrier proteins during transport

a

Histamine binds to the H1 G-protein-linked receptor to initiate the itchiness and airway constriction associated with an allergic response. If a mutation in the associated G-protein's alpha subunit prevented the hydrolysis of GTP how would the allergic response change? a. More severe allergic response compared to normal G-protein signaling b. Less severe allergic response compared to normal G-protein signaling c. No allergic response d. No change compared to normal G-protein signaling

a

How did Meselson and Stahl support Watson and Crick's double-helix model? a. They demonstrated that each strand serves as a template for synthesizing a new strand of DNA b. They showed that the DNA strands break and recombine without losing genetic material c. They proved that DNA maintains a double-helix structure while undergoing semi-conservative replication d. They demonstrated that conservative replication maintains the complementary base pairing of each DNA helix

a

Hydras and jellyfish both live in a freshwater lake that is slowly being acidified by the runoff from a chemical plant built upstream. Which population is predicted to be better able to cope with the changing environment? a. jellyfish b. hydra c. The populations will be equally able to cope d. Both populations will die

a

In any given species, there are at least how many types of aminoacyl tRNA synthetases? a. 20 b. 40 c. 100 d. 200

a

In each of the three systems, determine the state of entropy (low or high) when comparing the first and second: i. the instant that a perfume bottle is sprayed compared with 30 seconds later, ii. an old 1950s car compared with a brand new car, and iii. a living cell compared with a dead cell a. i. low, ii. high, iii. low b. i. low, ii. high, iii. high c. i. high, ii. low, iii. high d. i. high, ii. low, iii. low

a

In plant cells, the function of the lysosomes is carried out by __________. a. vacuoles b. peroxisomes c. ribosomes d. nuclei

a

In which direction does DNA replication take place? a. 5'-3' b. 3'-5' c. 5' d. 3'

a

Labrador retrievers' fur color is controlled by two alleles, E and B. Any dog with the ee__ genotype develops into a yellow lab, while B_E_ dogs become black labs and bbE_ dogs become chocolate labs. This is an example of _____. a. epistasis b. codominance c. incomplete dominance d. linkage

a

Mendel performed hybridizations by transferring pollen from the _______ of the male plant to the female ova a. anther b. pistil c. stigma d. seed

a

Phospholipids are important components of ________. a. the plasma membrane of cells b. the ring structure of steroids c. the waxy covering on leaves d. the double bond in hydrocarbon chains

a

Tay-Sachs disease is a genetic disorder that results in the destruction of neurons due to a buildup of sphingolipids in the cells. Which organelle is malfunctioning in Tay-Sachs? a. lysosome b. endoplasmic reticulum c. peroxisome d. mitochondria

a

The chromosomes become visible under a light microscope during which stage of mitosis? a. prophase b. prometaphase c. metaphase d. anaphase

a

The effect of high levels of ADP is to ________ in cellular respiration a. increase the activity of specific enzymes b. decrease the activity of specific enzymes c. have no effect on the activity of specific enzymes d. slow down the pathway

a

The energy currency used by cells is ________. a. ATP b. ADP c. AMP d. adenosine

a

The following chemical reactants produce the ester ethyl ethanoate (C4H8O2):C2H6O + CH3COOH. What type of reaction occurs to make ethyl ethanoate? a. condensation b. hydrolysis c. combustion d. acid-base reaction

a

The fusing of Golgi vesicles at the metaphase plate of dividing plant cells forms what structure? a. cell plate b. actin ring c. cleavage furrow d. mitotic spindle

a

The observable traits expressed by an organism are described as its ________. a. phenotype b. genotype c. alleles d. zygote

a

Three of the same species of plant are each grown under a different colored light for the same amount of time. Plant A is grown under blue light, Plant B is grown under green light, and Plant C is grown under orange light. Assuming the plants use only chlorophyll a and chlorophyll b for photosynthesis, what would be the predicted order of the plants from most growth to least growth? a. A, C, B b. A, B, C c. C, A, B d. B, A, C

a

What is the initial source of electrons for the chloroplast electron transport chain? a. Water b. Oxygen c. Carbon Dioxide d. NADPH

a

What is the primary function of carbohydrates attached to the exterior of cell membranes? a) identification of the cell b) flexibility of the membrane c) strengthening the membrane d) channels through membrane

a

What problem is faced by organisms that live in fresh water? a) Their bodies tend to take in too much water b) They have no way of controlling their tonicity c) Only salt water poses problems for animals that live in it d) Their bodies tend to lose too much water to their environment

a

What processing step enhances the stability of pre-tRNAs and pre-rRNAs? a. methylation b. nucleotide modification c. cleavage d. splicing

a

When acids are added to a solution, the pH should________. a. decrease b. increase c. stay the same d. cannot tell without testing

a

Which negative regulatory molecule can trigger cell suicide (apoptosis) if vital cell cycle events do not occur? a. p53 b. p21 c. retinoblastoma protein (Rb) d. cyclin-dependent kinase (Cdk)

a

Which of the following analogies best describes the induced-fit model of enzyme-substrate binding? a. a hug between two people b. a key fitting into a lock c. a square peg fitting through the square hole and around peg fitting through the round hole of a children's toy d. the fitting together of two jigsaw puzzle pieces

a

Which of the following components is not used by both plants and cyanobacteria to carry out photosynthesis? a. chloroplasts b. chlorophyll c. carbon dioxide d. water

a

Which of the following fermentation methods can occur in animal skeletal muscles? a. lactic acid fermentation b. alcohol fermentation c. mixed acid fermentation d. propionic fermentation

a

Which of the following is not a component of the endomembrane system? a. mitochondrion b. Golgi apparatus c. endoplasmic reticulum d. lysosome

a

Which of the following is not a functional group that can bond with carbon? a. sodium b. hydroxyl c. phosphate d. carbonyl

a

Which of the following is not an example of an energy transformation? a. turning on a light switch b. solar panels at work c. formation of static electricity d. none of the above

a

Which of the following is not true about enzymes? a. They increase ∆G of reactions b. They are usually made of amino acids c. They lower the activation energy of chemical reactions d. Each one is specific to the particular substrate(s) to which it binds

a

Which of the following molecules is likely to have the most potential energy? a. sucrose b. ATP c. glucose d. ADP

a

Which of the following organelles relies on exocytosis to complete its function? a) Golgi apparatus b) vacuole c) mitochondria d) endoplasmic reticulum

a

Which of the following structures is not a component of a photosystem? a. ATP synthase b. antenna molecule c. reaction center d. primary electron acceptor

a

Which plasma membrane component can be either found on its surface or embedded in the membrane structure? a) protein b) cholesterol c) carbohydrate d) phospholipid

a

Which statement correctly describes carbon fixation? a. the conversion of CO2 into an organic compound b. the use of RuBisCO to form 3-PGA c. the production of carbohydrate molecules from G3P d. the formation of RuBP from G3P molecules e. the use of ATP and NADPH to reduce CO2

a

Which type of bond represents a weak chemical bond? a. hydrogen bond b. atomic bond c. covalent bond d. nonpolar covalent bond

a

Many antibiotics inhibit bacterial protein synthesis. For example, tetracycline blocks the A site on the bacterial ribosome, and chloramphenicol blocks peptidyl transfer. What specific effect would you expect each of these antibiotics to have on protein synthesis? Tetracycline would directly affect: a. tRNA binding to the ribosome b. ribosome assembly c. growth of the protein chain Chloramphenicol would directly affect a. tRNA binding to the ribosome b. ribosome assembly c. growth of the protein chain

a c

Animals and plants both have diploid and haploid cells. How does the animal life cycle differ from the alternation of generations exhibited by plants?

a. In the haploid-dominant life cycle, the multicellular stage is haploid. The diploid stage is a spore that undergoes meiosis to produce cells that will divide mitotically to produce new multicellular organisms. Fungi have a haploid-dominant life cycle. b. In the diploid-dominant life cycle, the most visible or largest multicellular stage is diploid. The haploid stage is usually reduced to a single cell type, such as a gamete or spore. Animals, such as humans, have a diploid-dominant life cycle. c. In the alternation of generations life cycle, there are both haploid and diploid multicellular stages, although the haploid stage may be completely retained by the diploid stage. Plants have a life cycle with alternation of generations.

A reducing chemical reaction ________. a. reduces the compound to a simpler form b. adds an electron to the substrate c. removes a hydrogen atom from the substrate d. is a catabolic reaction

b

A scientist creates fruit fly larvae with a mutation that eliminates the exonuclease function of DNA pol III. Which prediction about the mutational load in the adult fruit flies is most likely to be correct? a. The adults with the DNA pol III mutation will have significantly more mutations than average b. The adults with the DNA pol III mutation will have slightly more mutations than average c. The adults with the DNA pol III mutation will have the same number of mutations as average d. The adults with the DNA pol III mutation will have fewer mutations than average

b

A scientist observes a mutation in the transmembrane region of EGFR that eliminates its ability to be stabilized by binding interactions during dimerization after ligand binding. Which hypothesis regarding the effect of this mutation on EGF signaling is most likely to be correct? a. EGF signaling cascades would be active for longer in the cell b. EGF signaling cascades would be active for a shorter period of time in the cell c. EGF signaling cascades would not occur d. EGF signaling would be unaffected

b

A scientist randomly mutates the DNA of a bacterium. She then sequences the bacterium's daughter cells, and finds that the daughters have many errors in their replicated DNA. The parent bacterium likely acquired a mutation in which enzyme? a. DNA ligase b. DNA pol II c. Primase d. DNA pol I

b

An organism's traits are determined by the specific combination of inherited _____. a. cells b. genes c. proteins d. chromatids

b

Attachment of the mitotic spindle fibers to the kinetochores is a characteristic of which stage of mitosis? a. prophase b. prometaphase c. metaphase d. anaphase

b

Chromosomes are duplicated during what stage of the cell cycle? a. G1 phase b. S phase c. prophase d. prometaphase

b

Dehydration synthesis leads to formation of a. monomers b. polymers c. water and polymers d. none of the above

b

From which component of the light-dependent reactions does NADPH form most directly? a. photosystem II b. photosystem I c. cytochrome complex d. ATP synthase

b

FtsZ proteins direct the formation of a _______ that will eventually form the new cell walls of the daughter cells a. contractile ring b. cell plate c. cytoskeleton d. septum

b

GTP or ATP is produced during the conversion of________. a. isocitrate into α-ketoglutarate b. succinyl CoA into succinate c. fumarate into malate d. malate into oxaloacetate

b

How do enhancers and promoters differ? a. Enhancers bind transcription factors to silence gene expression, while promoters activate transcription. b. Enhancers increase the efficiency of gene expression, but are not essential for transcription. Promoter recognition is essential to transcription initiation. c. Promoters bind transcription factors to increase the efficiency of transcription. Enhancers bind RNA polymerases to initiate transcription. d. There is no difference. Both are transcription factor-binding sequences in DNA.

b

How does PKC's signaling role change in response to growth factor signaling versus an immune response? a. PKC interacts directly with signaling molecules in both cascades, but only exhibits kinase activity during growth factor signaling b. PKC interacts directly with signaling molecules in growth factor cascades, but interacts with signaling inhibitors during immune signaling c. PKC amplifies growth factor cascades, but turns off immune cascades d. PKC is activated during growth factor cascades, but is inactivated during immune response cascades

b

How many photons does it take to fully reduce one molecule of NADP+ to NADPH? a. 1 b. 2 c. 4 d. 8

b

If a muscle cell of a typical organism has 32 chromosomes, how many chromosomes will be in a gamete of that same organism? a. 8 b. 16 c. 32 d. 64

b

Imagine a cell can perform exocytosis, but only minimal endocytosis. What would happen to the cell? a) The cell would secrete all its intracellular proteins b) The plasma membrane would increase in size over time c) The cell would stop expressing integral receptor proteins in its plasma membrane d) The cell would lyse

b

Imagine you are performing a cross involving seed color in garden pea plants. What F1 offspring would you expect if you cross true-breeding parents with green seeds and yellow seeds? Yellow seed color is dominant over green. a. 100 percent yellow-green seeds b. 100 percent yellow seeds c. 50 percent yellow, 50 percent green seeds d. 25 percent green, 75 percent yellow seeds

b

In humans, _____ are used to move a cell within its environment while _____ are used to move the environment relative to the cell a. cilia, pseudopodia b. flagella; cilia c. microtubules; flagella d. microfilaments; microtubules

b

Lactose is a disaccharide formed by the formation of a________ bond between glucose and ________. a. glycosidic; lactose b. glycosidic; galactose c. hydrogen; sucrose d. hydrogen; fructose

b

Meselson and Stahl's experiments proved that DNA replicates by which mode? a. conservative b. semi-conservative c. dispersive d. none of the above

b

Peroxisomes got their name because hydrogen peroxide is: a. used in their detoxification reactions b. produced during their oxidation reactions c. incorporated into their membranes d. a cofactor for the organelles' enzymes

b

Plant cell walls contain which of the following in abundance? a. starch b. cellulose c. glycogen d. lactose

b

Plants containing only chlorophyll bare exposed to radiation with the following wavelengths: 10nm (x-rays), 450nm (blue light), 670nm (red light), and 800nm (infrared light). Which plants harness the most energy for photosynthesis? a. X-ray irradiated plants b. Blue light irradiated plants c. Red light irradiated plants d. Infrared irradiated plants

b

Predict the end result if a chloroplast's light-independent enzymes developed a mutation that prevented them from activating in response to light. a. G3P accumulation b. ATP and NADPH accumulation c. Water accumulation d. Carbon dioxide depletion

b

The control of which enzyme exerts the most control on glycolysis? a. hexokinase b. phosphofructokinase c. glucose-6-phosphatase d. aldolase

b

The forked line and probability methods make use of what probability rule? a. test cross b. product rule c. monohybrid rule d. sum rule

b

The initial mechanism for repairing nucleotide errors in DNA is ________. a. mismatch repair b. DNA polymerase proofreading c. nucleotide excision repair d. thymine dimers

b

The mitotic spindles arise from which cell structure? a. centromere b. centrosome c. kinetochore d. cleavage furrow

b

The pea plants used in Mendel's genetic inheritance studies were diploid, with 14 chromosomes in somatic cells. Assuming no crossing over events occur, how many unique gametes could one pea plant produce? a. 28 b. 128 c. 196 d. 16,384

b

The process by which a cell engulfs a foreign particle is known as: a. endosymbiosis b. phagocytosis c. hydrolysis d. membrane synthesis

b

The α-helix and the β-pleated sheet are part of which protein structure? a. primary b. secondary c. tertiary d. quaternary

b

What do the electrons added to NAD+ do? a. They become part of a fermentation pathway b. They go to another pathway for ATP production c. They energize the entry of the acetyl group into the citric acid cycle d. They are converted to NADP

b

What is the function of a phosphatase? a. A phosphatase removes phosphorylated amino acids from proteins b. A phosphatase removes the phosphate group from phosphorylated amino acid residues in a protein c. A phosphatase phosphorylates serine, threonine, and tyrosine residues d. A phosphatase degrades second messengers in the cell

b

What is the ploidy of the most conspicuous form of most fungi? a. diploid b. haploid c. alternation of generations d. asexual

b

What property prevents the ligands of cell-surface receptors from entering the cell? a. The molecules bind to the extracellular domain b. The molecules are hydrophilic and cannot penetrate the hydrophobic interior of the plasma membrane c. The molecules are attached to transport proteins that deliver them through the bloodstream to target cells d. The ligands are able to penetrate the membrane and directly influence gene expression upon receptor binding

b

What structure is most important in forming the tetrads? a. centromere b. synaptonemal complex c. chiasma d. kinetochore

b

What transcripts will be most affected by low levels of α-amanitin? a. 18S and 28S rRNAs b. pre-mRNAs c. 5S rRNAs and tRNAs d. other small nuclear RNAs

b

Which feature of promoters can be found in both prokaryotes and eukaryotes? a. GC box b. TATA box c. octamer box d. -10 and -35 sequences

b

Which is one of the seven characteristics that Mendel observed in pea plants? a. flower size b. seed texture c. leaf shape d. stem color

b

Which of the following does the enzyme primase synthesize? a. DNA primer b. RNA primer c. Okazaki fragments d. phosphodiester linkage

b

Which of the following statements is not true? a. water is polar b. water stabilizes temperature c. water is essential for life d. water is the most abundant molecule in the Earth's atmosphere

b

Which of the following statements is true? a. acids and bases cannot mix together b. acids and bases will neutralize each other c. acids, but not bases, can change the pH of a solution d. acids donate hydroxide ions (OH-); bases donate hydrogen ions (H+)

b

Which statement about thylakoids in eukaryotes is not correct? a. Thylakoids are assembled into stacks b. Thylakoids exist as a maze of folded membranes c. The space surrounding thylakoids is called stroma d. Thylakoids contain chlorophyll

b

Which statement best describes the genetic content of the two daughter cells in prophase II of meiosis? a. haploid with one copy of each gene b. haploid with two copies of each gene c. diploid with two copies of each gene diploid with four copies of each gene

b

Which transport mechanism can bring whole cells into a cell? a) pinocytosis b) phagocytosis c) facilitated transport d) primary active transport

b

Why are ion channels necessary to transport ions into or out of a cell? a. Ions are too large to diffuse through the membrane b. Ions are charged particles and cannot diffuse through the hydrophobic interior of the membrane c. Ions do not need ion channels to move through the membrane d. Ions bind to carrier proteins in the bloodstream, which must be removed before transport into the cell

b

A farmer raises black and white chickens. To his surprise, when the first generation of eggs hatch all the chickens are black with white speckles throughout their feathers. What should the farmer expect when the eggs laid after interbreeding the speckled chickens hatch? a. All the offspring will be speckled b. 75% of the offspring will be speckled, and 25% will be black c. 50% of the offspring will be speckled, 25% will be black, and 25% will be white d. 50% of the offspring will be black and 50% of the offspring will be white

c

A gene that codes for a positive cell-cycle regulator is called a(n) _____. a. kinase inhibitor b. tumor suppressor gene c. proto-oncogene d. oncogene

c

A recessive trait will be observed in individuals that are________ for that trait. a. heterozygous b. homozygous or heterozygous c. homozygous d. diploid

c

A scientist notices that when she adds a small, water-soluble molecule to a dish of cells, the cells turn off transcription of a gene. She hypothesizes that the ligand she added binds to a(n) ______ receptor. a. Intracellular b. Hormone c. Enzyme-linked d. Gated ion channel-linked

c

A scientist pollinates a true-breeding pea plant with violet, terminal flowers with pollen from a true-breeding pea plant with white, axial flowers. Which of the following observations would most accurately describe the F2 generation? a. 75% violet flowers; 75% terminal flowers b. 75% white flowers in a terminal position c. 75% violet flowers; 75% axial flowers d. 75% violet flowers in an axial position

c

An allosteric inhibitor does which of the following? a. Binds to an enzyme away from the active site and changes the conformation of the active site, increasing its affinity for substrate binding b. Binds to the active site and blocks it from binding substrate c. Binds to an enzyme away from the active site and changes the conformation of the active site, decreasing its affinity for the substrate d. Binds directly to the active site and mimics the substrate

c

Chemiosmosis involves ________. a. the movement of electrons across the cell membrane b. the movement of hydrogen atoms across a mitochondrial membrane c. the movement of hydrogen ions across a mitochondrial membrane d. the movement of glucose through the cell membrane

c

Consider a cross to investigate the pea pod texture trait, involving constricted or inflated pods. Mendel found that the traits behave according to a dominant/recessive pattern in which inflated pods were dominant. If you performed this cross and obtained 650 inflated-pod plants in the F2 generation, approximately how many constricted-pod plants would you expect to have? a. 600 b. 165 c. 217 d. 468

c

Consider a pendulum swinging. Which type(s) of energy is/are associated with the pendulum in the following instances: i. the moment at which it completes one cycle, just before it begins to fall back towards the other end, ii. the moment that it is in the middle between the two ends, and iii. just before it reaches the end of one cycle (just before instant i.). a. i. potential and kinetic, ii. potential and kinetic, iii.kinetic b. i. potential, ii. potential and kinetic, iii. potential and kinetic c. i. potential, ii. kinetic, iii. potential and kinetic d. i. potential and kinetic, ii. kinetic iii. kinetic

c

During the second half of glycolysis, what occurs? a. ATP is used up b. Fructose is split in two c. ATP is made d. Glucose becomes fructose

c

Energy is stored long-term in the bonds of _____ and used short-term to perform work from a(n) _____molecule. a. ATP : glucose b. an anabolic molecule : catabolic molecule c. glucose : ATP d. a catabolic molecule : anabolic molecule

c

HER2 is a receptor tyrosine kinase. In 30 percent of human breast cancers, HER2 is permanently activated, resulting in unregulated cell division. Lapatinib, a drug used to treat breast cancer, inhibits HER2 receptor tyrosine kinase autophosphorylation (the process by which the receptor adds phosphates onto itself), thus reducing tumor growth by 50 percent. Besides autophosphorylation, which of the following steps would be inhibited by Lapatinib? a. Signaling molecule binding, dimerization, and the downstream cellular response b. Dimerization, and the downstream cellular response. c. The downstream cellular response d. Phosphatase activity, dimerization, and the downstream cellular response

c

How does the sodium-potassium pump make the interior of the cell negatively charged? a) by expelling anions b) by pulling in anions c) by expelling more cations than are taken in d) by taking in and expelling an equal number of cations

c

How many NADH molecules are produced on each turn of the citric acid cycle? a. one b. two c. three d. four

c

How many nucleotides are in 12 mRNA codons? a. 12 b. 24 c. 36 d. 48

c

If DNA of a particular species was analyzed and it was found that it contains 27 percent A, what would be the percentage of C? a. 27 percent b. 30 percent c. 23 percent d. 54 percent

c

If four molecules of carbon dioxide enter the Calvin cycle (four "turns" of the cycle), how many G3P molecules are produced and how many are exported? a. 4 G3P made, 1 G3P exported b. 4 G3P made, 2 G3P exported c. 8 G3P made, 1 G3P exported d. 8 G3P made, 4 G3P exported

c

If the sequence of the 5'-3' strand is AATGCTAC, then the complementary sequence has the following sequence: a. 3'-AATGCTAC-5' b. 3'-CATCGTAA-5' c. 3'-TTACGATG-5' d. 3'-GTAGCATT-5'

c

In a mating between two individuals that are heterozygous for a recessive lethal allele that is expressed in utero, what genotypic ratio (homozygous dominant: heterozygous: homozygous recessive) would you expect to observe in the offspring? a. 1:2:1 b. 3:1:1 c. 1:2:0 d. 0:2:1

c

In what important way does receptor-mediated endocytosis differ from phagocytosis? a) It transports only small amounts of fluid b) It does not involve the pinching off of membrane c) It brings in only a specifically targeted substance d) It brings substances into the cell, while phagocytosis removes substances

c

Mad cow disease is an infectious disease where one misfolded protein causes all other copies of the protein to begin misfolding. This is an example of a disease impacting ____ structure. a. primary b. secondary c. tertiary d. quaternary

c

Many farmers are worried about the decreasing genetic diversity of plants associated with generations of artificial selection and inbreeding. Why is limiting random sexual reproduction of food crops concerning? a. Mutations during asexual reproduction decrease plant fitness b. Consumers do not trust identical-appearing produce c. Larger portions of the plant populations are susceptible to the same diseases d. Spores are not viable in an agricultural setting

c

Many of the negative regulator proteins of the cell cycle were discovered in what type of cells? a. gametes b. cells in G0 c. cancer cells d. stem cells

c

Many viruses enter host cells through receptor-mediated endocytosis. What is an advantage of this entry strategy? a) The virus directly enters the cytoplasm of the cell b) The virus is protected from recognition by white blood cells c) The virus only enters its target host cell type d) The virus can directly inject its genome into the cell's nucleus.

c

Meiosis usually produces ________ daughter cells a. two haploid b. two diploid c. four haploid d. four diploid

c

Potassium has an atomic number of 19. What is its electron configuration? a. shells 1 and 2 are full, and shell 3 has nine electrons b. shells 1, 2 and 3 are full and shell 4 has three electrons c. shells 1, 2 and 3 are full and shell 4 has one electron d. shells 1, 2 and 3 are full and no other electrons are present

c

Separation of the sister chromatids is a characteristic of which stage of mitosis? a. prometaphase b. metaphase c. anaphase d. telophase

c

The RNA components of ribosomes are synthesized in the ________. a. cytoplasm b. nucleus c. nucleolus d. endoplasmic reticulum

c

The key components of desmosomes are cadherins and__________. a. actin b. microfilaments c. intermediate filaments d. microtubules

c

The monomers that make up proteins are called________. a. nucleotides b. disaccharides c. amino acids d. chaperones

c

The part of meiosis that is similar to mitosis is ________. a. meiosis I b. anaphase I c. meiosis II d. interkinesis

c

The principal force driving movement in diffusion is the__________. a) temperature b) particle size c) concentration gradient d) membrane surface area

c

The secretion of hormones by the pituitary gland is an example of _______________. a. autocrine signaling b. paracrine signaling c. endocrine signaling d. direct signaling across gap junctions

c

Water moves via osmosis _________. a) throughout the cytoplasm b) from an area with a high concentration of other solutes to a lower one c) from an area with a high concentration of water to one of lower concentration d) from an area with a low concentration of water to higher concentration

c

We call a molecule that binds up excess hydrogen ions in a solution a(n) ________. a. acid b. isotope c. base d. donator

c

What happens to the membrane of a vesicle after exocytosis? a) It leaves the cell b) It is disassembled by the cell c) It fuses with and becomes part of the plasma membrane d) It is used again in another exocytosis event

c

What is a likely evolutionary advantage of sexual reproduction over asexual reproduction? a. Sexual reproduction involves fewer steps b. There is a lower chance of using up the resources in a given environment c. Sexual reproduction results in variation in the offspring d. Sexual reproduction is more cost-effective

c

What is the effect of an inhibitor binding an enzyme? a. The enzyme is degraded b. The enzyme is activated c. The enzyme is inactivated d. The complex is transported out of the cell

c

What is the main prerequisite for clearance at the G2 checkpoint? a. cell has reached a sufficient size b. an adequate stockpile of nucleotides c. accurate and complete DNA replication d. proper attachment of mitotic spindle fibers to kinetochores

c

What phase of mitotic interphase is missing from meiotic interkinesis? a. G0 phase b. G1 phase c. S phase d. G2 phase

c

What property enables the residues of the amino acids serine, threonine, and tyrosine to be phosphorylated? a. They are polar b. They are non-polar c. They contain a hydroxyl group d. They occur more frequently in the amino acid sequence of signaling proteins

c

What two main products result from photosynthesis? a. oxygen and carbon dioxide b. chlorophyll and oxygen c. sugars/carbohydrates and oxygen d. sugars/carbohydrates and carbon dioxide

c

When viewing a specimen through a light microscope, scientists use ________ to distinguish the individual components of cells. a. a beam of electrons b. radioactive isotopes c. special stains d. high temperatures

c

Where in eukaryotic cells does the Calvin cycle take place? a. thylakoid membrane b. thylakoid lumen c. chloroplast stroma d. granum

c

Which complex is not involved in the establishment of conditions for ATP synthesis? a. photosystem I b. ATP synthase c. photosystem II d. cytochrome complex

c

Which eukaryotic cell-cycle event is missing in binary fission? a. cell growth b. DNA duplication c. karyokinesis d. cytokinesis

c

Which event contradicts the central dogma of molecular biology? a. Poly-A polymerase enzymes process mRNA in the nucleus b. Endonuclease enzymes splice out and repair damaged DNA c. Scientists use reverse transcriptase enzymes to make DNA from RNA d. Codons specifying amino acids are degenerate anduniversal

c

Which of the following are only in plant cells? a. gap junctions b. desmosomes c. plasmodesmata d. tight junctions

c

Which of the following is most likely to have the greatest concentration of smooth endoplasmic reticulum? a. a cell that secretes enzymes b. a cell that destroys pathogens c. a cell that makes steroid hormones d. a cell that engages in photosynthesis

c

Which of the following is not a true statement comparing prokaryotic and eukaryotic DNA replication? a. Both eukaryotic and prokaryotic DNA polymerases build off RNA primers made by primase b. Eukaryotic DNA replication requires multiple replication forks, while prokaryotic replication uses a single origin to rapidly replicate the entire genome c. DNA replication always occurs in the nucleus d. Eukaryotic DNA replication involves more polymerases than prokaryotic replication

c

Which of the following is not an extracellular matrix role of carbohydrates? a. protect an insect's internal organs from external trauma b. prevent plant cells from lysing after the plant is watered c. maintain the shape of a fungal spore d. provide energy for muscle movement

c

Which of the following is not true in regard to crossover? a. Spindle microtubules guide the transfer of DNA across the synaptonemal complex b. Nonsister chromatids exchange genetic material c. Chiasmata are formed d. Recombination nodules mark the crossover point

c

Which of the following sequences correctly lists in order the steps involved in the incorporation of a proteinaceous molecule within a cell? a. protein synthesis of the protein on the ribosome; modification in the Golgi apparatus; packaging in the endoplasmic reticulum; tagging in the vesicle b. synthesis of the protein on the lysosome; tagging in the Golgi; packaging in the vesicle; distribution in the endoplasmic reticulum c. synthesis of the protein on the ribosome; modification in the endoplasmic reticulum; tagging in the Golgi; distribution via the vesicle d. synthesis of the protein on the lysosome; packaging in the vesicle; distribution via the Golgi; tagging in the endoplasmic reticulum

c

Which of the following statements about quorum sensing is false? a. Autoinducer must bind to receptor to turn on transcription of genes responsible for the production of more autoinducer b. The receptor stays in the bacterial cell, but the autoinducer diffuses out c. Autoinducer can only act on a different cell: it cannot act on the cell in which it is made d. Autoinducer turns on genes that enable the bacteria to form a biofilm

c

___________ are changes to the order of nucleotides in a segment of DNA that codes for a protein. a. Proto-oncogenes b. Tumor suppressor genes c. Gene mutations d. Negative regulators

c

In the diagram below, protons are moved to one side of a semi-permeable gradient while a redox reaction occurs. This is an example of a ______________that is needed for ATP synthesis. (note: you must insert the entire correct name. No partial credit)

chemiosmotic gradient

1. Which of the following is the correct order of events in mitosis? a. Sister chromatids line up at the metaphase plate. The kinetochore becomes attached to the mitotic spindle. The nucleus reforms and the cell divides. Cohesin proteins break down and the sister chromatids separate. b. The kinetochore becomes attached to the mitotic spindle. Cohesin proteins break down and the sister chromatids separate. Sister chromatids lineup at the metaphase plate. The nucleus reforms and the cell divides. c. The kinetochore becomes attached to the cohesion proteins. Sister chromatids line up at the metaphase plate. The kinetochore breaks down and the sister chromatids separate. The nucleus reforms and the cell divides. d. The kinetochore becomes attached to the mitotic spindle. Sister chromatids line up at the metaphase plate. Cohesin proteins break down and the sister chromatids separate. The nucleus reforms and the cell divides.

d

A scientist notices that a cancer cell line fails to die when he adds an inducer of apoptosis to his culture of cells. Which hypothesis could explain why the cells fail to die? a. The cells have a mutation that prevents the initiation of apoptosis signaling b. The cells have lost expression of the receptor for the apoptosis-inducing ligand c. The cells overexpress a growth factor pathway that inhibits apoptosis d. All of the above

d

Active transport must function continuously because__________. a) plasma membranes wear out b) not all membranes are amphiphilic c) facilitated transport opposes active transport d) diffusion is constantly moving solutes in opposite directions

d

At which stage of meiosis are sister chromatids separated from each other? a. prophase I b. prophase II c. anaphase I d. anaphase II

d

Atoms that vary in the number of neutrons found in their nuclei are called ________. a. ions b. neutrons c. neutral atoms d. isotopes

d

Bacterial transformation is a major concern in many medical settings. Why might health care providers be concerned? a. Pathogenic bacteria could introduce disease-causing genes in non-pathogenic bacteria b. Antibiotic resistance genes could be introduced tone bacteria to create "superbugs" c. Bacteriophages could spread DNA encoding toxins to new bacteria d. All of the above

d

Cellulose and starch are examples of: a. monosaccharides b. disaccharides c. lipids d. polysaccharides

d

Congenital disorders of glycosylation are a growing class of rare diseases. Which organelle would be most commonly involved in the glycoprotein disorder portion of the group? a. RER b. ribosomes c. endosomes d. Golgi apparatus

d

DNA double helix does not have which of the following? a. antiparallel configuration b. complementary base pairing c. major and minor grooves d. uracil

d

Diseased animal cells may produce molecules that activate death cascades to kill the cells in a controlled manner. Why would neighboring healthy cells also die? a. The death molecule is passed through desmosomes b. The death molecule is passed through plasmodesmata c. The death molecule disrupts the extracellular matrix d. The death molecule passes through gap junctions

d

Each carbon molecule can bond with as many as ________ other atom(s) or molecule(s). a. one b. two c. six d. four

d

How are the NADPH and G3P molecules made during photosynthesis similar? a. They are both end products of photosynthesis b. They are both substrates for photosynthesis c. They are both produced from carbon dioxide d. They both store energy in chemical bonds

d

How do telophase I and telophase II differ during meiosis in animal cells? a. Cells remain diploid at the end of telophase I, but are haploid at the end of telophase II b. Daughter cells form a cell plate to divide during telophase I, but divide by cytokinesis during telophase II c. Cells enter interphase after telophase I, but not after telophase II d. Chromosomes can remain condensed at the end of telophase I, but decondense after telophase II

d

How does NF-κB induce gene expression? a. A small, hydrophobic ligand binds to NF-κB, activating it b. Phosphorylation of the inhibitor Iκ-B dissociates the complex between it and NF-κB, and allows NF-κB to enter the nucleus and stimulate transcription c. NF-κB is phosphorylated and is then free to enter the nucleus and bind DNA d. NF-κB is a kinase that phosphorylates a transcription factor that binds DNA and promotes protein production

d

How many different offspring genotypes are expected in a trihybrid cross between parents heterozygous for all three traits when the traits behave in a dominant and recessive pattern? How many phenotypes? a. 64 genotypes; 16 phenotypes b. 16 genotypes; 64 phenotypes c. 8 genotypes; 27 phenotypes d. 27 genotypes; 8 phenotypes

d

Human papillomavirus can cause cervical cancer. The virus encodes E6, a protein that binds p53. Based on this fact and what you know about p53, what effect do you think E6 binding has on p53 activity? a. E6 activates p53 b. E6 inactivates p53 c. E6 mutates p53 d. E6 binding marks p53 for degradation

d

Identical copies of chromatin held together by cohesin at the centromere are called _____. a. histones b. nucleosomes c. chromatin d. sister chromatids

d

If black and white true-breeding mice are mated and the result is all gray offspring, what inheritance pattern would this be indicative of? a. dominance b. codominance c. multiple alleles d. incomplete dominance

d

If the M checkpoint is not cleared, what stage of mitosis will be blocked? a. prophase b. prometaphase c. metaphase d. anaphase

d

If the allele encoding polydactyly (six fingers) is dominant why do most people have five fingers? a. Genetic elements suppress the polydactyl gene b. Polydactyly is embryonic lethal c. The sixth finger is removed at birth d. The polydactyl allele is very rare in the human population

d

In eukaryotes, what is the DNA wrapped around? a. single-stranded binding proteins b. sliding clamp c. polymerase d. histones

d

In which situation would passive transport not use a transport protein for entry into a cell? a) water flowing into a hypertonic environment b) glucose being absorbed from the blood c) an ion flowing into a nerve cell to create an electrical potential d) oxygen moving into a cell after oxygen deprivation

d

Prokaryotes depend on ________ to obtain some materials and to get rid of wastes. a. ribosomes b. flagella c. cell division d. diffusion

d

Quorum sensing is triggered to begin when ___________. a. treatment with antibiotics occurs b. bacteria release growth hormones c. bacterial protein expression is switched on d. a sufficient number of bacteria are present

d

Saturated fats have all of the following characteristics except: a. they are solid at room temperature b. they have single bonds within the carbon chain c. they are usually obtained from animal sources d. they tend to dissolve in water easily

d

The ABO blood groups in humans are expressed as the IA, IB, and i alleles. The IA allele encodes the A blood group antigen, IB encodes B, and i encodes O. Both A and B are dominant to O. If a heterozygous blood type A parent (IAi) and a heterozygous blood type B parent (IBi) mate, one quarter of their offspring will have AB blood type (IAIB) in which both antigens are expressed equally. Therefore, ABO blood groups are an example of: a. multiple alleles and incomplete dominance b. codominance and incomplete dominance c. incomplete dominance only d. multiple alleles and codominance

d

The AUC and AUA codons in mRNA both specify isoleucine. What feature of the genetic code explains this? a. complementarity b. nonsense codons c. universality d. degeneracy

d

The building blocks of nucleic acids are ________. a. sugars b. nitrogenous bases c. peptides d. nucleotides

d

The ends of the linear chromosomes are maintained by a. helicase b. primase c. DNA pol d. telomerase

d

The energy released by the hydrolysis of ATP is____ a. primarily stored between the alpha and beta phosphates b. equal to −57 kcal/mol c. harnessed as heat energy by the cell to perform work d. providing energy to coupled reactions

d

The experiments by Hershey and Chase helped confirm that DNA was the hereditary material on the basis of the finding that: a. radioactive phage were found in the pellet b. radioactive cells were found in the supernatant c. radioactive sulfur was found inside the cell d. radioactive phosphorus was found in the cell

d

The first level of DNA organization in a eukaryotic cell is maintained by which molecule? a. cohesion b. condensin c. chromatin d. histone

d

Three different bacteria species have the following consensus sequences upstream of a conserved gene. Species A Species B Species C -10 TAATAAT TTTAAT TATATT -35 TTGACA TTGGCC TTGAAA Order the bacteria from most to least efficient initiation of gene transcription. a. A > B > C b. B > C > A c. C > B > A d. A > C > B

d

Unpacking of chromosomes and the formation of a new nuclear envelope is a characteristic of which stage of mitosis? a. prometaphase b. metaphase c. anaphase d. telophase

d

What is removed from pyruvate during its conversion into an acetyl group? a. oxygen b. ATP c. B vitamin d. carbon dioxide

d

What is the combination of an electrical gradient and a concentration gradient called? a) potential gradient b) electrical potential c) concentration potential d) electrochemical gradient

d

Which molecule is a Cdk inhibitor that is controlled by p53? a. cyclin b. anti-kinase c. Rb d. p21

d

Which molecule must enter the Calvin cycle continually for the light-independent reactions to take place? a. RuBisCO b. RuBP c. 3-PGA d. CO2

d

Which of the following comparisons or contrasts between endergonic and exergonic reactions is false? a. Endergonic reactions have a positive ∆G and exergonic reactions have a negative ∆G b. Endergonic reactions consume energy and exergonic reactions release energy c. Both endergonic and exergonic reactions require a small amount of energy to overcome an activation barrier d. Endergonic reactions take place slowly and exergonic reactions take place quickly

d

Which of the following components is not involved during the formation of the replication fork? a. single-strand binding proteins b. helicase c. origin of replication d. ligase

d

Which of the following do not play a role in intracellular movement? a. microfilaments and intermediate filaments b. microfilaments and microtubules c. intermediate filaments and microtubules d. only intermediate filaments

d

Which of the following events does not occur during some stages of interphase? a. DNA duplication b. organelle duplication c. increase in cell size d. separation of sister chromatids

d

Which of the following is both in eukaryotic and prokaryotic cells? a. nucleus b. mitochondrion c. vacuole d. ribosomes

d

Which of the following is surrounded by two phospholipid bilayers? a. the ribosomes b. the vesicles c. the cytoplasm d. the nucleoplasm

d

Which of the following organisms is a prokaryote? a. amoeba b. influenza A virus c. charophyte algae d. E. coli

d

Which of the following situations does not follow the Law of Independent Assortment? a. A blond man and a brunette woman produce three offspring over time, all of who have blond hair b. A white cow crossed with a brown bull produces roan cattle c. Mating a hog with a sow produces six female piglets d. Men are more likely to experience hemophilia than women

d

Which of the following statements is true? a. In photosynthesis, oxygen, carbon dioxide, ATP, and NADPH are reactants. GA3P and water are products b. In photosynthesis, chlorophyll, water, and carbon dioxide are reactants. GA3P and oxygen are products c. In photosynthesis, water, carbon dioxide, ATP, and NADPH are reactants. RuBP and oxygen are products d. In photosynthesis, water and carbon dioxide are reactants. GA3P and oxygen are products

d

Which pre-mRNA processing step is important for initiating translation? a. poly-A tail b. RNA editing c. Splicing d. 7-methylguanosine cap

d

Which protein is a positive regulator that phosphorylates other proteins when activated? a. p53 b. retinoblastoma protein (Rb) c. cyclin d. cyclin-dependent kinase (Cdk)

d

Which subunit of the E. coli polymerase confers specificity to transcription? a. α b. β c. β' d. σ

d

Which type of life cycle has both a haploid and diploid multicellular stage? a. asexual life cycles b. most animal life cycles c. most fungal life cycles d. alternation of generations

d


Ensembles d'études connexes

Anthropology Test Questions Chapter 6

View Set

Sgt. Pepper's Lonely Hearts Club Band Album

View Set

08 Zeiten - Konjunktiv II 03 - irrealer Wunsch/ Träume

View Set

Life Insurance Premiums, Proceeds and Beneficiaries

View Set